Jan 23

Download as pdf or txt
Download as pdf or txt
You are on page 1of 80

PARADISE WEEKLY MODEL TEST – CEE MD/MS BASED MAGH 09

PARADISE INSTITUTE & STUDY CENTER


CEE MD/MS BASED MODEL TEST
(Tuesday, Magh 9, 2080)
SCAN THE QR CODE AND JOIN THE COMMUNITY OF PG ASPIRANTS IN VIBER
- Discuss with fellow aspirants.
- Get notices about MD/MS Entrances
- Get notices about mock tests, results and offers
- Ask confusing questions to the experts
- Scroll the groups, and learn.

Questions Answers Explanations (QAE)


MEDICINE
1. A 55-year-old man is noted to have moderately severe congestive heart failure with impaired systolic function. Which of
the following drugs would most likely lower his risk of mortality?
a. ACE inhibitors
b. Loop diuretics
c. Digoxin
d. Aspirin

Ans: a
Ref: Harrison 20th Edition, Jessup M, Brozena S. Heart failure. N Engl J Med. 2003;348:2007-2018.
Angiotensin-converting enzyme inhibitors and beta-blockers decrease the risk of mortality when used to treat CHF with impaired
systolic function. For this reason, these agents are the initial choice to treat CHF. They both prevent and can even, in some
circumstances, reverse the cardiac remodeling.

2. A 55-year-old man is noted to have congestive heart failure and states that he is comfortable at rest but becomes dyspneic
even with walking to the bathroom. On ECHO, he is noted to have an LVEF 47%. Which of the following is the more
accurate description of this patient’s condition?
a. Diastolic dysfunction
b. Systolic dysfunction
c. Dilated cardiomyopathy
d. Pericardial disease

Ans: a
Ref: Lejemtel TH, Sonnenblick EH, Frishman WH. Diagnosis and management of heart failure. In: Fuster V, Alexander RX,
O’Rourke RA, eds. Hurst’s the Heart. 10th ed. New York, NY: McGraw-Hill; 2001:6.
When the ejection fraction exceeds 40%, there is likely diastolic dysfunction, with stiff ventricles. The stiff thickened
ventricles do not accept blood very readily. This patient has symptoms with mild exertion that are indicative of functional class III.
The worst class is level IV, manifested as symptoms at rest or with minimal exertion. ACE inhibitors are important agents in patients
with diastolic dysfunction.

3. A 48-year-old man presents with progressive visual loss and is found to have fulminant cytomegalovirus (CMV) infection
of the retina. He also has recurrent oral candidiasis. A deficiency of which of the following immune components is most
likely to cause these problems?
a. Phagocyte deficiency
b. Complement deficiency
c. T-lymphocyte deficiency
d. Neutrophil deficiency

Ans: c
Paradise Institute & Self Study Centre Page 1
PARADISE WEEKLY MODEL TEST – CEE MD/MS BASED POUSH 09

Ref: Davidson’s Principles and practice of medicine, 23rd Ed, Page 73


This is a typical presentation of T-lymphocyte deficiency, which can cause increased susceptibility to viral, fungal and mycobacterial
infections. The most common cause of T-lymphocyte deficiency worldwide is HIV infection.

4. A78-year-old is noted to have fever and chills, decreased mentation, tachycardia, and right lower quadrant abdominal
tenderness, and guarding. Which of the following is the most likely diagnosis?
a. Ruptured diverticulitis
b. Meningitis
c. Ruptured appendicitis
d. Ischemic bowel

Ans : c
Ref: Harrison’s Principles of Internal Medicine. 18th ed. New York, NY: McGraw-Hill; 2012: 2502-2510.
The most common cause of an acute abdomen at any age is appendicitis.

5. A 65-year-old man with colon cancer on chemotherapy presents with a fever and headache of 3-day duration. An LP is
performed, and Gram stain reveals gram-positive rods. Which of the following therapies is most likely to treat the
organism?
a. Vancomycin
b. Metronidazole
c. Ampicillin
d. Ceftriaxone

Ans: c
Ref: Harrison’s Principles of Internal Medicine. 18th ed. New York, NY: McGraw-Hill; 2012:1211-1219.
Listeria monocytogenes is a gram-positive rod that causes approximately 10% of all cases of meningitis. It is more common in the
elderly and in other patients with impaired cell-mediated immunity, such as patients on chemotherapy. It is also more common in
neonates. It is not sensitive to cephalosporins, and specific therapy with ampicillin must be instituted if the suspicion for this disease
is high.

Paradise Institute & Self Study Centre CALL/VIBER: 9818723799 Page 2


PARADISE WEEKLY MODEL TEST – CEE MD/MS BASED POUSH 09

6. A traveler on an expedition to the Annapurna base camp (altitude 4130 m) becomes unwell on the third day of the trip.
He initially complains of breathlessness, dry cough, and fatigue. The cough subsequently becomes productive of bloody
sputum and on examination he is found to be hypoxic with crepitations in both lung fields. What is the likeliest diagnosis?
a. Acute mountain sickness
b. Pulmonary embolism
c. High-altitude pulmonary oedema
d. Acute respiratory distress syndrome

Ans: c
Ref: Davidson’s Principles and practice of medicine, 23rd Ed, Page 169
• This is a fairly classical description of high-altitude pulmonary oedema (HAPE). The most important treatments are
immediate descent and administration of oxygen.
• High-altitude pulmonary oedema (HAPE) is a life-threatening condition that usually occurs in the first 4 days after ascent
above 2500 m. Unlike HACE, HAPE may occur de novo without the preceding signs of AMS. Presentation is with symptoms
of dry cough, exertional dyspnoea and extreme fatigue. Later, the cough becomes wet and sputum may be blood-stained.
• Tachycardia and tachypnoea occur at rest and crepitations may often be heard in both lung fields. There may be profound
hypoxaemia, pulmonary hypertension and radiological evidence of diffuse alveolar oedema. It is not known whether the
alveolar oedema is a result of mechanical stress on the pulmonary capillaries associated with the high pulmonary arterial
pressure, or an effect of hypoxia on capillary permeability. Reduced arterial oxygen saturation is not diagnostic but is a
marker for disease progression.

7. A patient with which of the following conditions requires antimicrobial pro- phylaxis before dental surgery?
a. Atrial septal defect
b. Mitral valves prolapse without mitral regurgitation
c. Previous coronary artery bypass graft
d. Previous infective endocarditis

Ans: a
Ref: Davidson’s Principles and practice of medicine, 23rd Ed, Page 715
Paradise Institute & Self Study Centre CALL/VIBER: 9818723799 Page 3
PARADISE WEEKLY MODEL TEST – CEE MD/MS BASED POUSH 09

• Ascorbic acid is the most active reducing agent in the aqueous phase of living tissues and is involved in intracellular electron
transfer. It takes part in the hydroxylation of proline and lysine in protocollagen to hydroxyproline and hydroxylysine in mature
collagen. It is very easily destroyed by heat, increased pH and light, and is very soluble in water; hence many traditional cooking
methods reduce or eliminate it. Claims that high-dose vitamin C improves immune function (including resistance to the common
cold) and cholesterol turnover remain unsubstantiated
• Elderly people who are socially and geographically isolated may not have ready access to fresh fruit and vegetables, the most
abundant source of vitamin C (ascorbic acid). This vitamin is important in the formation of collagen; when it is deficient, defective
collagen formation impairs healing of wounds, causes capillary haemorrhage and reduces platelet adhesiveness (normal platelets
are rich in ascorbate). This manifest clinically as perifollicular and petechial haemorrhages, easy bruising, swollen gums which
bleed easily, and occasionally gastrointestinal bleeding. Ascorbic acid is also a powerful reducing agent, which converts the ferric
form of iron found in the diet to the more readily absorbable ferrous form, thereby increasing body iron
absorption.

8. A 65-year-old patient in the ICU appears to be making an excellent recovery from pneumonia but then deteriorates
acutely. Which of the following diagnoses is the most likely cause of his deterioration?
a. Major haemorrhage
b. Pulmonary embolism
c. Septic shock (pre-volume load)
d. Acute left ventricular failure secondary to myocardial infarction

Ans: d
Ref: Mylonakis E, Calderwood SB. Infective endocarditis in adults. N Engl J Med. 2001;345:1318-1330.
Prior endocarditis damages valvular surfaces, and these patients are at increased risk for reinfection during a transient bacteremia, as
may occur during dental procedures or some other GI or genitourinary tract procedures. All of the other conditions mentioned have
a negligible risk of endocarditis, the same as in the general population, and antibiotic prophylaxis is not recom- mended by the
American Heart Association.

9. In which of the following conditions is continuous positive airways pressure (CPAP) usually most effective?
a. Acute severe asthma
b. Pulmonary oedema secondary to left ventricular failure (LVF)
c. Exacerbation of chronic obstructive pulmonary disease (COPD)
Paradise Institute & Self Study Centre CALL/VIBER: 9818723799 Page 4
PARADISE WEEKLY MODEL TEST – CEE MD/MS BASED POUSH 09

d. Neuromuscular disorder leading to respiratory failure

Ans: b
Ref: Davidson’s Principles and practice of medicine, 23rd Ed, Page 202
CPAP is an effective treatment to improve oxygenation in LVF. It does not increase ventilation (the volume of air shifted in and out
of the lungs) so it is not a good treatment for disorders characterised by hypoventilation,
such as COPD, narcotic drug overdose and neuromuscular disorders. These patients may benefit from bi-level positive airway
pressure, which does support ventilation. Drug overdose patients are not normally treated in this way because they are at risk of
aspiration pneumonitis, so the airway must be protected with a cuffed endotracheal tube. CPAP is contraindicated in acute severe
asthma as it worsens hyperexpansion, so decreasing ventilation, which may be fatal.

10. A 42-year-old woman is being treated with infliximab for rheumatoid arthritis. After 6 months of therapy, she develops
persistent fever, weight loss, and night sweats, and TB is suspected. Which of the following is the most likely location of
the tuberculosis?
a. Middle and lower lung zones
b. Pleural space
c. Apical segment of the upper lung lobes
d. Cervical or supraclavicular lymph nodes

Ans: c
Ref: Harrison’s Prin- ciples of Internal Medicine. 18th ed. New York, NY: McGraw-Hill; 2012:1340-1359.
Reactivation tuberculosis (in this case, likely triggered by infliximab) usually involves the apical aspects of the lungs. Primary TB
infection most often affects the middle and lower lung zones. Lymphadenitis and pleural infection are the most common
extrapulmonary sites of TB, but they are less common than pulmonary TB.
✓ Reactivation pulmonary tuberculosis most commonly presents radio- graphically with infiltrates or nodules in the apical and
posterior segments of the upper lobes.
✓ Patients with a positive tuberculin skin test and no clinical or radiographic evidence of active disease are said to have
latent tuberculosis infection; they can be treated with isoniazid to reduce their lifetime risk of develop- ing reactivation
tuberculosis.
✓ Individuals with active tuberculosis should be initiated on multidrug therapy, such as isoniazid, rifampin, pyrazinamide,
and ethambutol.
✓ Pyridoxine (vitamin B6) is usually added to antituberculosis medications to prevent peripheral neuropathy.

11. A 35-year-old woman presents with a 24-hour history of watery diarrhoea. She has opened her bowels nine times
since the onset of her symptoms. You diagnose gastroenteritis after learning that the patient and her family all ate at
a new restaurant and the rest of her family have had similar problems. The most appropriate management is:
a. Oral rehydration advice, anti-emetics and discharge home
b. Oral antibiotic therapy and discharge home
c. Admission for intravenous antibiotic therapy
d. No treatment required

Ans: a
Ref: Davidson’s Principles and practice of medicine, 23rd Ed, Page 228
• Gastroenteritis is usually a self-limiting disease that often does not require pharmacological therapy. The mainstay of
treatment is to advise patients to increase oral fluid intake to compensate for the water lost from diarrhoea and vomiting.
• However, in some circumstances, where severe dehydration secondary to profuse diarrhoea exists (leading to confusion and
hypotension) patients may warrant admission for intravenous fluid rehydration.
• Antibiotic therapy is usually not indicated for gastroenteritis unless a bacterial organism has been isolated. The fact that the
patient’s observations are within the normal range and she is not systemically unwell, the most appropriate answer here
would be to advise the patient on oral fluid rehydration and prescribe antiemetics (e.g. metoclopramide), followed by
discharge.

Paradise Institute & Self Study Centre CALL/VIBER: 9818723799 Page 5


PARADISE WEEKLY MODEL TEST – CEE MD/MS BASED POUSH 09

12. Which of the following is most likely to cause restrictive cardiomyopathy?


a. Endomyocardial fibrosis
b. Viral myocarditis
c. Beriberi (thiamine deficiency)
d. Doxorubicin therapy

Ans: a
Ref: Wynne J, Braunwald E. Cardiomyopathy and myocarditis. In: Longo DL, Fauci AS, Kasper DL, et al., eds. Harrison’s
Principles of Internal Medicine. 18th ed. New York, NY: McGraw-Hill; 2012:1951-197
Endomyocardial fibrosis is an etiology of restrictive cardiomyopathy, common in developing countries, that is associated with
eosinophilia. The other disease processes mentioned are causes of dilated cardiomyopathy.

13. In CML, there occurs a reciprocal translocation between:


a. Chromosomes 2 and 13
b. Chromosomes 11 and 14
c. Chromosomes 9 and 22
d. Chromosomes 11 and 22

Ans: c
Ref: Harrison 20th Edition, Page No: 454

Paradise Institute & Self Study Centre CALL/VIBER: 9818723799 Page 6


PARADISE WEEKLY MODEL TEST – CEE MD/MS BASED POUSH 09

14. Which of the following are the most likely physical examination findings in a patient with emphysema?
a. Diffuse expiratory wheezing.
b. Clubbing of the fingers
c. Bibasilar inspiratory crackles with increased jugular venous pressure (JVP)
d. Inspiratory stridor

Ans: a
Ref: Harrison’s Principles of Internal Medicine. 18th ed. New York, NY: McGraw-Hill; 2012:1380-1388.
COPD is characterized by chronic airway obstruction, with most airflow resistance occurring in small airways of the lower respiratory
tract, producing expiratory wheezing. Inspiratory stridor would occur with upper airway, usually extrathoracic, obstruction. Clubbing
is not generally a feature of COPD and should prompt investigation for another disease process such as a broncho- genic carcinoma.
Crackles, elevated JVP, and an S3 are signs of congestive heart failure.

15. A man is admitted to your ICU with acute liver failure. The following criteria must be met to fulfil a diagnosis of acute
liver failure:
a. Illness duration <28 days.
b. Presence of chronic liver disease.
c. Jaundice.
d. Coagulopathy with an INR >1.5.

Ans: d
Ref: Trotter JF. Practical management of acute liver failure in the intensive care unit. Curr Opin Crit Care 2009; 15: 163-7
Acute liver failure (ALF) is a rare but multisystem condition of various aetiologies. In the developed world, paracetamol overdose is
the most common cause of ALF; however, worldwide, viral aetiologies such as hepatitis A, B and E and seronegative hepatitis are
more prevalent. The management of such patients on the ICU may increase their survival chances before the next steps (e.g. liver

Paradise Institute & Self Study Centre CALL/VIBER: 9818723799 Page 7


PARADISE WEEKLY MODEL TEST – CEE MD/MS BASED POUSH 09

transplantation) are taken. The diagnosis of ALF is made using the specific criteria listed below. However that the clinical picture is
often dominated by coagulopathy and encephalopathy.

16. A 24-year-old patient has 4 days history of fever and cold. Crept are heard in bilateral lower chest regions.
Neutrophil is the dominant cell type in alveolar space. Which of the following is the pathological phase of his
pneumonia?
a. Edema
b. Red hepatization
c. Gray hepatization
d. Resolution

Ans: c
Ref: Ref: Harrison 20th Edition, Page No: 909
Classic pneumonia evolves through a series of pathologic changes.
• The initial phase is one of edema, with the presence of a proteinaceous exudate— and often of bacteria—in the alveoli.
• This phase is rarely evident in clinical or autopsy specimens because of the rapid transition to the red hepatization phase.
The presence of erythrocytes in the cellular intraalveolar exudate gives this second stage its name, but neutrophil influx
is more important with regard to host defense. Bacteria are occasionally seen in pathologic specimens collected during
this phase.
• In the third phase, gray hepatization, no new erythrocytes are extravasating, and those already present have been lysed
and degraded. The neutrophil is the predominant cell, fibrin deposition is abundant, and bacteria have disappeared. This
phase corresponds with successful containment of the infection and improvement in gas exchange.
• In the final phase, resolution, the macrophage reappears as the dominant cell type in the alveolar space, and the debris of
neutrophils, bacteria, and fibrin has been cleared, as has the inflammatory response.

17. A 54-year-old woman is noted to have cervical cancer and presents with significant vaginal bleeding with a
hemoglobin level of 7 g/dL. Her left leg is swollen, which on Doppler investigation reveals a DVT. Which of the
following is the best treatment for the thrombus?
a. Fractionated subcutaneous heparin
b. Subcutaneous unfractionated heparin
c. Oral warfarin (Coumadin)
d. Vena cava filter

Ans: d
Ref: Goldhaber SZ. Deep venous thrombosis and pulmonary thromboembolism. In: Longo DL, Fauci AS, Kasper DL,
et al., eds. Harrison’s Principles of Internal Medicine. 18th ed. New York, NY: McGraw-Hill; 2012:2170-2177.

Paradise Institute & Self Study Centre CALL/VIBER: 9818723799 Page 8


PARADISE WEEKLY MODEL TEST – CEE MD/MS BASED POUSH 09

Cervical cancer with significant vaginal bleeding is a relative contraindication for anticoagulation. Thus, a vena cava filter is
the most appropriate choice in this patient.

18. The preferred fibrinolytic regimen in PE is:


a. Recombinant tissue plasminogen activator (tPA)
b. Streptokinase
c. Urokinase
d. Alteplase

Ans: a
Ref: Harrison 19th Edition, Page No: 1636
• The preferred fibrinolytic regimen is 100 mg of recombinant tissue plasminogen activator (tPA) administered as a
continuous peripheral intravenous infusion over 2 h.
• The sooner thrombolysis is administered, the more effective it is.
• However, this approach can be used for at least 14 days after the PE has occurred.
• Contraindications to fibrinolysis include intracranial disease, recent surgery, and trauma. The overall major bleeding rate
is about 10%, including a 1–3% risk of intracranial hemorrhage.
• Careful screening of patients for contraindications to fibrinolytic therapy is the best way to minimize bleeding risk.

Surgery
19. A 45-year-old man complains of a dragging discomfort in the left side of his scrotum. He noticed an irregular
lump in his scrotum separate from the testis. He thinks he has had this for many years but only recently he noticed
discomfort in the lump. On examination he has a bunch of veins in the scrotum and the testis feels normal. On
lying down the veins disappear. Abdominal examination in the left renal area does not show any abnormality.
Likely condition is;
a. Acute epidydimo-orchitis
b. Testicular tumour
c. Torsion of the testis
d. Varicocele

Ans: d
Ref: Bailey and Love’s Short Practice of Surgery 27th Edition, Page No: 1031
This patient has a left varicocele. As it is on the left side, it is important to exclude a left renal carcinoma. Rarely a tumour
thrombus might spread along the left renal vein, blocking the left testicular vein causing the varicocele. Abdominal
examination of the left lumbar area and loin is mandatory. In the vast majority the cause is not known and 90% occur on the
left side, probably because of the termination of the left testicular vein into the left renal vein at right angles. The association
of a varicocele with subfertility or oligospermia is anything but proven and
improvement of semen quality is not an indication for surgical intervention. Indication for intervention for a varicocele is
significant discomfort. Under the circumstances embolization of the testicular veins is the treatment of choice. If that fails,
operation by an extraperitoneal route for surgical ligation of the testicular vein through an incision in the iliac fossa is
undertaken. Recurrence does occur even after surgical ligation.

20. A 32-year-old man presented to ER with a multiple gunshot wound to the abdomen. His SBP is 70 and his HR
140 bpm. On CT, Heterogenous enhancement is seen and he is taken directly to the operating room for an
exploratory laparotomy. A large, actively bleeding liver laceration is found. A Pringle maneuver is performed as
part of the procedure to control his bleeding, which means compression of:
a. Portal vein, hepatic vein, and hepatic artery
b. Portal vein, hepatic artery, and cystic artery
c. Portal vein and hepatic artery
d. Portal vein, hepatic artery, and CBD

Ans: d

Paradise Institute & Self Study Centre CALL/VIBER: 9818723799 Page 9


PARADISE WEEKLY MODEL TEST – CEE MD/MS BASED POUSH 09

Ref: Bailey and Love’s Short Practice of Surgery 27th Edition, Page No: 374
Pringle’s maneuver is occlusion of the porta hepatis. The portal vein, hepatic artery, and the CBD are the structures of the
porta hepatis.
The operative management of liver injuries can be summarized as ‘the four Ps’:
▪ Push;
▪ Pringle;
▪ Plug;
▪ Pack.

21. A 32-year-old woman is noted to have nipple discharge. She is concerned about the possible association with
breast cancer. Which of the following etiologies of nipple discharge most likely increases the risk of breast cancer?
a. Fibrocystic changes
b. Blood-tinged discharge
c. Intraductal papilloma
d. Diffuse papillomatosis

Ans: d
Ref: Leitch AM, Ashfaq R. Discharges and secretions of the nipple. In: Bland KI, Copeland III EM, eds. The Breast.
Comprehensive Management of Benign and Malignant Disease. Philadelphia, PA: Saunders Elsevier; 2009:59-83.
Papillary breast lesions encompass a spectrum of masses, which pathologically demonstrate fronds attached to the inner
mammary duct wall by a fibrovascular core with both epithelial and myoepithelial cells. Although not frankly malignant,
papillary disease is associated with an increased risk of invasive breast cancer. The epithelial component of papillary lesions
can vary, resulting in metaplasia, hyperplasia, atypical intraductal hyperplasia, or carcinoma in-situ.
Juvenile papillary pathology is associated with a strong family history of breast cancer. Multiple papillomas may be
highly susceptible to malignant transformation, likely due to their slightly different cytologic architecture, and are also
associated with a higher risk (3.0) of hyperplasia, atypia, DCIS, sclerosis adenosis, and radial scar. Papillomatosis with atypia
has a 7.0 higher relative risk of developing breast carcinoma.

22. Which of the following statements regarding female urethra is true?


a. The bladder neck is the most important entity for urinary continence.
b. Fowler’s syndrome is best treated by urethral dilatation.
c. Urethral caruncle is premalignant.
d. Urethral diverticulum is more common in women than men

Ans: d
Ref: Bailey and Love’s Short Practice of Surgery 27th Edition, Page No: 1479
• Urethral prolapse, also called urethrocele, often occurs with cystoceles. The condition occurs in later life and is partly
due to trauma at childbirth. Weakening of the tissue that hold the urethra in place cause it to move, producing pressure
on the vagina and resulting in prolapse of the anterior vaginal wall.
• Urethral diverticulum is much more common in women. It occurs due to rupture of a distended urethral gland or
urethral injury during labour. It causes local pain and repeated cystitis. Diagnosis is by MRI or transvaginal US.
Excision is the treatment that should be carried out with great caution, as it might damage the urethral sphincter.
• Bladder neck has hardly any role in maintaining urinary continence, which is maintained by the external urethral
sphincter, which consists of striated muscle that envelops the whole length of the female urethra. Fowler’s syndrome
causes urinary retention. It is caused by abnormal myotonic discharge in the striated urethral sphincter. Treatment is
by intermittent self-catheterisation, as urethral dilatation is ineffective. Urethral caruncle is a pedunculated granuloma
and is not premalignant.

23. Which of the following tumor characteristics of non–small cell lung cancer does contraindicate pulmonary
resection?
a. Involvement of the parietal pleura by a 3-cm tumor
b. 2.5-cm tumor with a single 1.5-cm peribronchial node on the ipsilateral side

Paradise Institute & Self Study Centre CALL/VIBER: 9818723799 Page 10


PARADISE WEEKLY MODEL TEST – CEE MD/MS BASED POUSH 09

c. 3-cm left lower lobe tumor with left pleural effusion that contains malignant cells
d. 5-cm tumor involving the right upper and middle lobe

Ans: c
Ref: Nason KS, Maddaus MA, Luketich JD. Chest wall, lung, mediastinum, and pleura. In: Brunicardi FC, Andersen
DK, Billiar TR, et al, eds. Schwartz’s Principles of Surgery. 9th ed. New York, NY: McGraw-Hill; 2010:513-590.
Pleural effusion associated with lung cancer is common and is not always produced by the tumor; however, in this
case the presence of malignant cell indicates extension of the tumor into the pleural space that precludes operative treatment.
Isolated parietal pleura involvement by tumor extension can be treated with en bloc resection of the lung, involved pleura,
and chest wall. A 2.5-cm primary lung cancer with peribronchial or ipsilateral hilar nodal involvement constitutes stage IIA
non–small cell carcinoma, which is best treated by pulmonary resection with chemotherapy + radiation therapy. A patient
with tumor involvement of the right upper and middle lobe can still be successfully treated by lung resection if his/her
pulmonary reserve is adequate. Isolated lung cancer involving the mainstem bronchus could be completely resected by a
pneumonectomy.

24. A 32-year-old man was involved in a road-traffic accident in which he sustained polytrauma, chief among which
was fracture of pelvis with fractured femoral shaft. After stabilization according to the ATLS protocol, it became
apparent that he has a distended bladder and unable to pass urine. The most likely scenario is;
a. Bladder rupture
b. Rupture of prostatic urethra
c. Rupture of bulbous urethra
d. Rupture of membranous urethra

Ans: d
Ref: Bailey and Love’s Short Practice of Surgery 27th Edition, Page No: 1481
Membranous urethral rupture is almost always associated with a fractured pelvis, except when it simulates an extra-peritoneal
rupture of the bladder. The diagnosis is suspected by the history and confirmed by an ascending urethrogram. The immediate
treatment is a suprapubic cystostomy. At times a retropubic haematoma and extravasation might need to be drained. Definitive
treatment must always be by the urologist
who will do a delayed repair and urethroplasty.

Paradise Institute & Self Study Centre CALL/VIBER: 9818723799 Page 11


PARADISE WEEKLY MODEL TEST – CEE MD/MS BASED POUSH 09

25. Gold standard test for Insulinoma is:


a. 72 hr fasting test
b. Plasma insulin levels
c. C-peptide levels
d. Low glucose levels < 30 mg/dl

Ans: a
Ref: Ref: Sabiston Surgery 20th Edition, Page No: 952
The gold standard for diagnosis of Insulinoma is the 72-hour monitored fast. However, in cases where hypoglycemic episodes
are observed and the aforementioned laboratory testing can be obtained, a monitored fast is unnecessary.
Insulinoma is the most common functioning PNET.
Whipple’s triad:
I. Neuroglycopenic symptoms consistent with hypoglycemia
II. Low plasma glucose concentration measured when symptoms were present, and
III. Relief of symptoms with administration of glucose

26. A 40-year-old unrestrained man was the driver of a car that crashed into a tree when his car apparently veered
off the road. He was brought to the emergency department, and after his initial resuscitation and evaluation, he is
found to have multiple superficial scalp lacerations, a left subdural hematoma with no associated midline ship and
with a GCS of 14, a 60% left pneumothorax, and left tibia and fibular fractures that are associated with
diminished pedal pulses. Which of the following is the most appropriate sequence of prioritization for this
patient’s injuries?
a. Brain injury, pneumothorax, lower extremity injuries, and facial lacerations
b. Pneumothorax, lower extremity injuries, brain injuries, and facial lacerations
c. Brain injury, lower extremity injuries, pneumothorax, and facial lacerations
d. Pneumothorax, brain injury, lower extremity injuries, and facial lacerations

Ans: d
Ref: Committee on Trauma of the American College of Surgeons. Initial assessment and management. In: Advanced
Trauma Life Support Program for Doctors. 7th ed. Chicago, IL: Committee on Trauma of the American College of
Surgeons; 2004.
The treatment prioritization for patients with multiple injuries should always consider life-threatening injuries before
injuries that may compromise qualities of life and are non–life-threatening. In this patient, the pneumo- thorax needs to be
addressed as part of the ABCs. The subdural hematoma is potentially life threatening, but with the patient’s initial GCS of 14
this is not likely to pose a threat to his life currently. The lower extremity fractures are serious and may be compromising his
lower extremity circulation, but this poses no threat to his life. Similarly, the facial laceration repair has the lowest priority
among all his injuries.

27. A middle-aged patient presents with intermittent dysphagia. On evaluation distal bird beaking was
noticed in Barium study. On manometry, simultaneous multipeaked contractions of high amplitude
(>120 mm Hg) was seen. Diagnosis is:
a. Hiatal hernia
b. Scleroderma
c. Achalasia cardia
d. Diffuse esophagus spasm

Ans: d
Ref: Ref: Sabiston Surgery 20th Edition, Page No: 1015
• Diffuse Esophageal Spasm is a poorly understood hypermotility disorder of the esophagus.
• The basic pathology is related to a motor abnormality of the esophageal body that is most notable in the lower two
thirds of the esophagus.
Paradise Institute & Self Study Centre CALL/VIBER: 9818723799 Page 12
PARADISE WEEKLY MODEL TEST – CEE MD/MS BASED POUSH 09

• Muscular hypertrophy and degeneration of the branches of the vagus nerve in the esophagus have been observed. As
a result, the esophageal contractions are repetitive, simultaneous, and of high amplitude.
• The clinical presentation of DES is typically that of chest pain and dysphagia. These symptoms may be related to
eating or exertion and may mimic those of angina. Patients will complain of a squeezing pressure in the chest that
may radiate to the jaw, arms, and upper back.
• The symptoms are often pronounced during times of heightened emotional stress. Regurgitation of esophageal
contents and saliva is common but acid reflux is not. However, acid reflux can aggravate the symptoms, as can cold
liquids.
• The diagnosis of DES is made by radiographic and manometric studies.
• The classic picture of the corkscrew esophagus or pseudodiverticulosis on an esophagram is caused by the presence
of tertiary contractions and indicates advanced disease.
• A distal bird beak narrowing of the esophagus and normal peristalsis can also be noted.
• The classic manometry findings in DES are simultaneous multipeaked contractions of high amplitude (>120 mm Hg)
or long duration (>2.5 seconds).

28. A newborn male was brought to the hospital for evaluation of vomiting after each feeds. True statements about
hypertrophic pyloric stenosis are all except:
a. Present at 4 weeks
b. First born male is commonly affected.
c. Ramstedt operation is done.
d. Visible peristalsis is always seen.

Ans: d
Ref: Ref: Sabiston Surgery 20th Edition, Page No: 1869
Hypertrophic Pyloric Stenosis
• Disease of newborns, with an incidence of 1 in 300 to 900 live births.
• It is most common between the ages of 2 and 8 weeks.
• Boys are affected four times more often than girls, with first-born male infants being at highest risk.
• Hypertrophy of the circular muscle of the pylorus results in constriction and obstruction of the gastric outlet, leading
to nonbilious, projectile emesis, loss of hydrochloric acid with the onset of hypokalemic hypochloremic metabolic
alkalosis, and dehydration.
• Although the exact cause of HPS remains unknown, a lack of nitric oxide synthase in pyloric tissue has been
implicated.

Paradise Institute & Self Study Centre CALL/VIBER: 9818723799 Page 13


PARADISE WEEKLY MODEL TEST – CEE MD/MS BASED POUSH 09

• Infants with HPS generally present with progressively worsening nonbilious emesis. Over time, the emesis becomes
more frequent, forceful, and projectile in nature. On occasion, visible gastric peristalsis may be observed as a wave
of contractions from the left upper quadrant to the epigastrium.
• Shortly after emesis, infants usually crave additional feedings.
• A plain abdominal radiograph can show an enlarged gastric gas bubble.
• Palpation of the pyloric “olive” tumor in the epigastrium by an experienced examiner is pathognomonic for HPS. If
the olive is confirmed, no additional diagnostic testing is necessary. When the olive is not appreciated, an ultrasound
study should be obtained.
• Pyloric muscle thickness of more than 3 to 4 mm or a pyloric length greater than 15 to 18 mm in the presence of
functional gastric outlet obstruction is diagnostic.
• With an equivocal clinical presentation, an upper GI contrast study may be useful to evaluate for other causes of
vomiting.
• The Ramstedt pyloromyotomy remains the standard operation for HPS. Preoperatively, it is imperative that the infant
be fully resuscitated with IV fluids to establish an adequate urine output and to restore normal electrolyte balance.
• If not, there is a high risk for postoperative apnea because the infant with metabolic alkalosis has a propensity to
compensate by retaining respiratory carbon dioxide. Thus, the serum bicarbonate level needs to be normalized before
surgery, at least to a value of less than 30 mEq/L.

29. A mother with a week-old baby presents with complains of pain and redness in right breast. Which of the
following is false regarding her condition?
a. Infection caused mostly by Staph. aureus
b. Retracted nipple predisposes.
c. Feeding of affected side can be advised if mother manages.
d. Fluctuation is early sign of abscess formation.

Ans: d
Ref: Bailey and Love’s Short Practice of Surgery 27th Edition, Page No: 866
▪ The given condition is most likely lactational mastitis.
▪ Most are caused by S. aureus and, if hospital acquired, are likely to be penicillin resistant
▪ Although ascending infection from a sore and cracked nipple may initiate the mastitis, in many cases the lactiferous
ducts will first become blocked by epithelial debris leading to stasis; this theory is supported by the relatively high
incidence of mastitis in women with a retracted nipple
During the cellulitic stage, the patient should be treated with an appropriate antibiotic, such as flucloxacillin or coamoxiclav.
Feeding from the affected side may continue if the patient can manage. Support of the breast, local heat and analgesia will
help to relieve pain
The presence of pus can be confirmed with needle aspiration, and the pus should be sent for bacteriological culture.
In contrast to the majority of localised infections, fluctuation is a late sign

30. Which of the following is an accurate statement regarding testicular seminomas?


a. Fertility following treatment is generally excellent.
b. Orchiectomy is never indicated for treatment.
c. Pain is the most common presentation.
d. Seminomas are sensitive to radiation therapy

Ans: a
Ref: Sabiston Textbook of Surgery. 18th ed
Seminomas are sensitive to radiation therapy; therefore, extratesticular extension of the disease such as disease involving the
inguinal, iliac, and periaortic lymph nodes can be treated with radiation therapy following radical orchiectomy. Needle biopsy
is contraindicated for patients with testicular masses that are suspicious for testicular cancers. Following radiation therapy,
patients generally have significant reduction in fertility; therefore, the option of sperm banking should be discussed with
patients prior to treatment.

Paradise Institute & Self Study Centre CALL/VIBER: 9818723799 Page 14


PARADISE WEEKLY MODEL TEST – CEE MD/MS BASED POUSH 09

31. Bowel can get strangulated in all of the following space except:
a. Rectouterine pouch
b. lleocolic recess
c. Paraduodenal recess
d. Omental bursa

Ans: a
Ref: Ref: Bailey and Love 27th Edition, Page No: 1282
• The following are potential sites of internal herniation (all are rare):
• The foramen of Winslow;
• A defect in the mesentery;
• A defect in the transverse mesocolon;
• Defects in the broad ligament;
• Congenital or acquired diaphragmatic hernia;
• Duodenal retroperitoneal fossae – left paraduodenal and right duodenojejunal;
• Caecal/appendiceal retroperitoneal fossae – superior, inferior and retrocaecal;
• Intersigmoid fossa.

32. A 38-year-old woman whose mother has just undergone treatment for breast cancer is asking about how this
affects her and what can be done to lessen her chances of having the disease. Which of the following is the least
significant risk factor for breast cancer?
a. BRCA1 mutation
b. early menarche
c. Advancing age
d. Dietary fat intake

Ans: d
Ref: Schwartz’s Principles of Surgery, 11th Ed , Pg 350
Studies have failed to demonstrate a correlation between diet and breast cancer risk. Age is the most common risk factor.
Another important risk factor is family history in a first-degree relative or presence of a genetic mutation such as BRCA1 or
2, which can be inherited through either the maternal or the paternal side of the family. Other risk factors include excessive
estrogen exposure, obesity, alcohol use, hormone replacement, ionizing radiation, and a history of a prior breast cancer or
abnormal breast biopsy (LCIS or atypical hyperplasia).

33. A 78-year-old male presented to the hospital for regular checkup. On evaluation, “Bouveret sign” was
positive with fullness over the right lower region. The likely condition is:
a. Appendicitis
b. Acute pancreatitis
c. Metastatic gastric carcinoma
d. Colonic obstruction

Ans: d
Ref: Ref: Geriatric Assessment: Abdominal and Rectal Examination by Mark E. Williams, MD in Medscape
Name Clinical Feature
Bouveret sign Colonic obstruction with right lower fullness
Courvoisier Jaundice and a palpable gallbladder suggesting cancer of the head of the pancreas
sign
Strauss sign A mass in the pouch of Douglas on rectal examination suggesting metastatic gastric carcinoma

Paradise Institute & Self Study Centre CALL/VIBER: 9818723799 Page 15


PARADISE WEEKLY MODEL TEST – CEE MD/MS BASED POUSH 09

Tansini sign A scaphoid abdomen suggesting primary gastric or pancreatic cancer, whereas protuberance suggests
metastatic bowel disease
Troisier sign Visible enlargement of the left supraclavicular fossa

34. A 10-years old female who used to use the swimming pool regularly, comes with a three-day history of vomiting,
fever and abdominal pain. O/E, abdominal tenderness and guarding are present. The liver dullness is not
obliterated. Likely diagnosis is:
a. Gangrenous intussusceptions
b. Perforation
c. Spontaneous biliary peritonitis
d. Primary peritonitis

Ans: d
Ref: Bailey and Love’s Short Practice of Surgery 27th Edition, Page No: 1053
Primary peritonitis is an infection of the peritoneal cavity not directly related to other intra-abdominal
abnormalities. On percussion replacement of liver dullness with resonant sound is suggestive of
pneumoperitoneum.
Spontaneous bacterial peritonitis-
• Acute bacterial infection of the peritoneum without obvious focal source
o occurs in up to 20% of patients with ascites
o high recurrence rate
o most commonly from a single organism
▪ Escherichia coli
▪ Klebsiella pneumoniae
▪ Streptococcus pneumoniae
• Associated conditions
o alcohol cirrhosis / heart failure/ Budd-Chiari /nephrosis
o systemic lupus erythematosus
o flank dullness
• Diagnostic testing
• peritoneal fluid analysis: diagnostic paracentesis or via peritoneal catheter
• WBCs: ≥ 500 cells/ µL
• neutrophil count ≥ 250 cells/µL
• Gram stain and culture
• pH / lactate / serum ascites albumin gradient (SAAG) > 1.1

35. A child presents with congenital torticollis. True about the condition is?
a. Head is tilted towards and rotated away from tight sternocleidomastoid muscle
b. Head is tilted and rotated away from tight sternocleidomastoid muscle
c. Head is tilted and rotated towards tight sternocleidomastoid muscle
d. Head is tilted away from and rotated towards tight sternocleidomastoid muscle

Ans: a
Ref: Bailey and Love’s Short Practice of Surgery 27th Edition, Page No: 582
Torticollis
▪ In torticollis the head is tilted towards and rotated away from the tight sternocleidomastoid muscle.
▪ Congenital torticollis is usually secondary to intrauterine moulding but may present with fixed sternocleidomastoid
contracture or with a palpable ‘tumour’ within the muscle.
▪ There is a strong correlation with DDH.
▪ Most cases resolve with stretching but persistent cases develop facial asymmetry and require surgical release of the
sternocleidomastoid at one or both ends.

Paradise Institute & Self Study Centre CALL/VIBER: 9818723799 Page 16


PARADISE WEEKLY MODEL TEST – CEE MD/MS BASED POUSH 09

▪ Acquired torticollis is less common and may be caused by gastro-oesophageal reflux, posterior fossa tumour/other
regional abnormality, inflammation/infection, ocular problems or atlantoaxial rotatory subluxation

36. A 62-year-old man with diabetes returns to the ED after 3 days after undergoing incision and drainage of a
perirectal abscess. The patient com- plains of fever and malaise. Evaluation of the perirectal area reveals an open,
draining wound with a 20-cm area of surrounding induration, erythema, localized areas of blister formation, and
skin necrosis. The infection has extended to involve the perineum, scrotum, and anterior abdomen. Which of the
following is most likely represented by the process in this patient?
a. Fournier gangrene
b. Clostridial gas gangrene
c. NSTI caused by group A β-hemolytic Streptococcus
d. Polymicrobial synergistic NSTI

Ans: d
Ref: Bulger EM. Necrotizing skin and soft tissue infection. In: Cameron JL, Cameron AM, eds. Current Surgical
Therapy. 10th ed. Philadelphia, PA: Elsevier Saunders; 2011:662-665.
Necrotizing soft-tissue infections (NSTI) encompass a diverse disease process characterized by extensive, rapidly
progressive soft tissue inflammation and necrosis. NSTIs are increasingly more common and continue to be associated with
a fulminant course and high mortality rates. These infections comprise a spectrum of diseases ranging from necrosis of the
skin to life-threatening infections involving the fascia and muscle with systemic toxicity. They vary in predisposing and
causative factors, anatomic location, offending bacteria, and tissue level of involvement.

Pediatrics
37. During a routine physical examination, a solitary thyroid nodule is palpated in asymptomatic 10-year-old child
of the following, the MOST likely cause is
a. thyroid carcinoma
b. thyroid adenoma
c. thyroid abscess
d. thyroid cyst

Ans: a
Ref: Ref: Nelson Textbook of Pediatrics, Elsevier, 21st Edition, Pg 11478
In children with a solitary nodule, about 30-40% has a carcinoma, 20-30% has an adenoma, and the remainder will have
thyroid abscess, thyroid cyst, multinodular goiter, Hashimoto thyroiditis, subacute thyroiditis, or nonthyroidal neck mass.

38. A 5-year-old girl is referred to a pediatric rheumatologist with a 4-week history of mild swelling and decreased
range of motion in the left knee and right elbow. She is afebrile and appears otherwise well. Positive findings on
which of the following evaluations will be most helpful in establishing her diagnosis?
a. Arthrocentesis
b. Complete blood count
c. Computerized tomographic scan of the involved joints
d. Slit-lamp examination of her eyes.

Ans: d
Ref: Rudolph’s Pediatrics. 22nd ed.
JIA is the most common cause of uveitis in children. Uveitis onset may be insidious and may be the only initial
manifestation of JIA. The disease is more common in young girls. Slit-lamp findings include band keratopathy, posterior
synechiae, and cataracts. Children with JIA should have periodic slit-lamp examinations to detect eye disease early.
Consideration may be given to obtaining the tests suggested in the other answer choices, but positive results on these tests
are unlikely to be specific for JIA.

39. A 19-year-old adolescent female presents for an evaluation of lethargy. Her father notes that recently she has
looked pale. She eats a regular diet and has no significant past medical history. Her menses are regular and have
Paradise Institute & Self Study Centre CALL/VIBER: 9818723799 Page 17
PARADISE WEEKLY MODEL TEST – CEE MD/MS BASED POUSH 09

not been excessive. During the last few years, she has been working at seafood restaurant but is increasingly tired
and unable to complete all her work. Her CBC reveals a megaloblastic anemia. Which of the following is the next
appropriate study?
a. Folate level
b. Stool for rotavirus
c. Iron level
d. Stool for ova and parasites

Ans: d
Ref: Nelson Textbook of Pediatrics, Elsevier, 20th Edition, Page No: 513
The fish tapeworm Diphyllobothrium latum uses vitamin B12 for growth and egg production as many as one million eggs per
day may be produced.
The parasite also inactivates the vitamin B12–intrinsic factor complex,
inhibiting absorption in the terminal ileum. The fish tapeworm is the longest tapeworm to infect humans, sometimes growing
to more than 10 m in length. Most infestations are asymptomatic, with megaloblastic anemia occurring in 2% to 9% of
tapeworm infections. Risk factors include eating raw or undercooked fish. Eggs have a unique morphology and are easily
found in stool samples.

40. A term male is delivered vaginally to a 22-year-old mother. Immediately after birth he is noted to have a
scaphoid abdomen, cyanosis, and respiratory distress. Heart sounds are heard on the right side of the chest,
and the breath sounds seem to be diminished on the left side. Which of the following is the most appropriate
next step in his resuscitation?
a. Administer IV bicarbonate.
b. Initiate bag-and-mask intubation.
c. Initiate chest compressions immediately
d. Intubate with an endotracheal tube.

Ans: d
Ref: Newborn resuscitation. In: McMillan JA, Feigin RD, DeAngelis CD, Jones MD, eds. Oski’s Pediatrics: Principles
and Practice. 4th ed. Philadelphia, PA: Lippincott Williams & Wilkins; 2006:207-213.
The case describes diaphragmatic hernia. As a result of herniated bowel contents into the chest, these children often
have pulmonary hypoplasia. Bag-and-mask ventilation will cause accumulation of bowel gas (which is located in the chest)
and further respiratory compromise. Therefore, endotracheal intubation is the best course of action.

41. An infant girl is born via spontaneous vaginal delivery at 28-week gestation because of an incompetent
cervix. Which of the following features of her clinical course in the neonatal intensive care unit (ICU) is most
likely to correlate with her clinical outcome 5 years from now?
a. Administration of surfactant

Paradise Institute & Self Study Centre CALL/VIBER: 9818723799 Page 18


PARADISE WEEKLY MODEL TEST – CEE MD/MS BASED POUSH 09

b. Apnea of prematurity
c. Grade IV intraventricular hemorrhage
d. Retinopathy of prematurity stage 1 on initial ophthalmologic examination

Ans: a
Ref: Nelson Textbook of Pediatrics, Elsevier, 21st Edition
Intraventricular hemorrhage is a complication in preterm infants. It is associated with seizures, hydrocephalus, and
periventricular leukomalacia. A grade IV bleed involves the brain parenchyma, put- ting this child at higher risk for
neurodevelopmental handicap.

42. A case of neuroblastoma with limping and irritability associated with bone and bone marrow
metastases is known as:
a. Horner syndrome
b. Hutchinson syndrome
c. Opsoclonus-myoclonus-ataxia syndrome
d. Kerner-Morrison syndrome

Ans: b
Ref: Ref: Nelson pediatrics, 20th Edition, Page No: 2461

43. Adrenal production of androgen with development of underarm odor and faint genital hair (adrenarche) may
occur as early as;
a. 4 yr
b. 8. 5 yr
c. 6 yr
d. 7 yr

Ans: c
Ref: Nelson Textbook of Pediatrics, Elsevier, 21st Edition, PDF ref 4310
• Puberty is the biologic transition from childhood to adulthood. Pubertal changes include the appearance of the
secondary sexual characteristics, increase in height, change in body composition, and development of reproductive
capacity.
• Adrenal production of androgen, mainly dehydroepiandrosterone sulfate (DHEAS), may occur as early as 6 yr of age,
with development of underarm odor and faint genital hair (adrenarche). Maturation of the gonadotropin releasing
hormone (GnRH) pulse generator is among the earliest neuroendocrine changes associated with the onset of puberty.

Paradise Institute & Self Study Centre CALL/VIBER: 9818723799 Page 19


PARADISE WEEKLY MODEL TEST – CEE MD/MS BASED POUSH 09

44. A 10-year-old boy has a history of recurrent sinusitis and multiple episodes of pneumonia. A sweat electrolyte
test result is within the normal range. Your differential diagnosis now includes atopy, primary ciliary dyskinesia,
and which of the following?
a. Tuberculosis
b. Chronic granulomatous disease
c. Cystic fibrosis
d. Severe combined immunodeficiency

Ans: c
Ref: Nelson Textbook of Pediatrics, Elsevier, 21st Edition
CF cannot be ruled out (false-negative test results); the sweat chloride test should be repeated and/or other CF diagnostic
modalities considered. Bronchiectasis and chronic sinusitis are characteristic of ciliary dyskinesia syndromes. If associated
with visceral situs inversus, the diagnosis of Kartagener disease is given. Sinusitis is not a common complaint among patients
with tuberculosis, chronic granulomatous disease, or coccidioidomycosis. In addition to severe pulmonary infections, chronic
diarrhea and wasting dominate the clinical picture in children with severe combined immunodeficiency.

45. The treatment of choice for a 3-year-old boy with Wiskott-Aldrich syndrome is-
a. splenectomy
b. use of killed vaccines
c. administration of steroids
d. trauma bone marrow or cord blood transplantation

Ans: d
Ref: Nelson Textbook of Pediatrics, Elsevier, 21st Edition, Pg Ref 4690
• Wiskott-Aldrich syndrome is an X-linked recessive disorder characterized byatopic dermatitis, thrombocytopenic
purpura with normal-appearing megakaryocytes but small defective platelets, and susceptibility to infection.
• The Wiskott-Aldrich syndrome protein (WASP) binds CDC42H2 and Rac, members of the Rho family of guanosine
triphosphatases.
• Patients often have prolonged bleeding from the circumcision site or bloody diarrhea during infancy. The
thrombocytopenia is not initially caused by antiplatelet antibodies. Atopic dermatitis and recurrent infections usually
develop during the 1st yr of life. Streptococcus pneumoniae and other bacteria having polysaccharide capsules cause
otitis media, pneumonia, meningitis, and sepsis. Later, infections with agents such as P. jiroveci and the herpesviruses
become more frequent. Infections, bleeding, and EBV-associated malignancies are major causes of death. The
predominant immunoglobulin pattern is a low serum level of IgM, elevated IgA and IgE, and a normal or slightly low
IgG concentration.
• Good supportive care includes appropriate nutrition, routine IVIG, use of killed vaccines, aggressive management of
eczema and associated cutaneous infections, platelet transfusion for serious bleeding episodes, splenectomy if a
transplant is not going to be done, and high-dose IVIG with systemic steroids for autoimmune complications. Bone
marrow or cord blood transplantation is the treatment of choice and is usually curative.

46. The following conditions can cause eosinophilia EXCEPT


a. malaria
b. filariasis
c. giardiasis
d. ascariasis

Ans: c
Ref: Nelson Textbook of Pediatrics, Elsevier, 21st Edition
Eosinophilia are caused by tissue-invasive as helminth infections; ascariasis filariasis, amebiasis, malaria, and scabies
whereas entrobius vermicularis and Giardia lamblia are solely intraluminal in gastrointestinal tract.

Paradise Institute & Self Study Centre CALL/VIBER: 9818723799 Page 20


PARADISE WEEKLY MODEL TEST – CEE MD/MS BASED POUSH 09

47. A mother brings to the clinic her 4-year-old son who began complaining of right knee pain 2 weeks ago, is
limping slightly, is fatigued, and has had a fever to 100.4°F (38°C). Which of the following laboratory tests is most
important?
a. Antinuclear antibodies
b. Complete blood count (CBC) with differential and platelets
c. Rheumatoid factor
d. Sedimentation rate

Ans: b
Ref: Nelson Textbook of Pediatrics, Elsevier, 21st Editionß
This child has symptoms of JRA and leukemia. The CBC with differential and platelets is the best initial screening test. The
leukocyte and platelet counts are normal to increased in JRA, and no blast cells are present. Frequently, blast cells are found
on the peripheral smear with ALL. The child in the question ultimately may require a bone marrow aspiration.

48. A 35-week-gestation infant is delivered via cesarean section because of macrosomia and fetal distress. The
mother has class D pregestational diabetes (insulin-dependent, with vascular disease); her hemoglobin A1C is 20%
(normal 8%). This infant is at risk for birth asphyxia, cardiac septal hypertrophy, polycythemia, and which of the
following?
a. Congenital dislocated hip
b. Dacryostenosis
c. Hyaline membrane disease
d. Hyperglycemia

Ans: c
Ref: Nelson Textbook of Pediatrics, Elsevier, 21st Edition
Infants born to mothers with poorly controlled diabetes are at risk for respiratory distress syndrome (surfactant deficiency) at
later gestational ages than seen in infants born to mothers who do not have diabetes.

Obstetrics & Gynecology


49. Piskacek’s sign is seen in:
a. Cervix
b. Vagina
c. Uterus
d. Ovary

Ans: c
Ref: Dutta Obstetrics 8th Edition, Page No: 74, 75
Jacquemier’s or Chadwick’s sign: It is the dusky hue of the vestibule and anterior vaginal wall visible at about 8th
week of pregnancy. The discoloration is due to local vascular congestion.
Vaginal sign:
(a) Apart from the bluish discoloration of the anterior vaginal wall
(b) The walls become softened and (c) Copious non-irritating mucoid discharge appears at 6th week (d) There is
increased pulsation, felt through the lateral Fornices at 8th week called Osiander’s sign.
Cervical signs: (a) Cervix becomes soft as early as 6th week (Goodell’s sign), a little earlier in multiparae. The
pregnant cervix feels like the lips of the mouth, while in the non-pregnant state, like that of tip of the nose. (b) On
speculum examination, the bluish discoloration of the cervix is visible. It is due to increased vascularity.
Uterine signs:
▪ Size, shape and consistency: The uterus is enlarged to the size of hen’s egg at 6th week, size of a cricket ball
at 8th week and size of a fetal head by 12th week. The pyriform shape of the non-pregnant uterus becomes
globular by 12 weeks. There may be asymmetrical enlargement of the uterus if there is lateral implantation.
This is called Piskacek’s sign where one half is firmer than the other half. As pregnancy advances, symmetry

Paradise Institute & Self Study Centre CALL/VIBER: 9818723799 Page 21


PARADISE WEEKLY MODEL TEST – CEE MD/MS BASED POUSH 09

is restored. Th e pregnant uterus feels soft and elastic.


▪ Hegar’s sign: It is present in two-thirds of cases. It can be demonstrated between 6 and 10 weeks, a little
earlier in multiparae. Th is sign is based on the fact that: (1) upper part of the body of the uterus is enlarged
by the growing fetus (2) lower part of the body is empty and extremely soft and (3) the cervix is comparatively
firm. Because of variation in consistency, on bimanual examination (two fingers in the anterior fornix and the
abdominal fingers behind the uterus), the abdominal and vaginal fingers seem to appose below the body of
the uterus. Examination must be gentle to avoid the risk of abortion.
▪ Palmer’s sign: Regular and rhythmic uterine contraction can be elicited during bimanual examination as early
as 4– 8 weeks. Palmer in 1949, first described it and it is a valuable sign when elicited.

50. A 23-year-old woman presents for her postpartum visit and contraception management. She delivered by
spontaneous vaginal delivery 6 weeks ago and is breastfeeding. After reviewing her history and performing
physical examination, you discuss the various methods of contraception with the patient. She opts for depot
medroxyprogesterone acetate (DepoProvera). Which of the following is a disadvantage of Depo-Provera?
a. Impairment of lactation
b. Increased risk of hepatic cancer
c. Irreversible bone loss
d. Prolonged anovulation

Ans: d
Ref: Schorge, pp 116-117
Depot medroxyprogesterone acetate (Depo-Provera) is a highly effective contraception. Its effectiveness is
comparable to or even better than pills. Its mechanisms of action include: ovulation suppression, cervical mucus thickening,
and decidualization of endometrium making it unfavorable for implantation. It has no impairment of lactation, and iron
deficiency anemia is less likely due to amenorrhea which develops in 80% of users. Its principal disadvantages are irregular
bleeding and prolonged anovulation which results in delayed return of fertility after discontinuation of the medication. Weight
gain is often attributed to depot medroxyprogesterone, but conclusive evidence is lacking. Cervical and hepatic cancers do
not appear to be increased and ovarian and endometrial cancers are decreased. Loss of bone mineral density is one concern,
but this loss is reversible after discontinuation of the medication.

51. Which of the following is not obstetric significance of Plane of Least Pelvic Dimension?
a. It is the widest plane in the pelvis
b. It is at this plane that the internal rotation of the fetal head occurs
c. It is a landmark used for pudendal nerve block analgesia
d. This level of ischial spines indicate Station ‘O’

Ans: a
Ref: Dutta Obstetrics 8th Edition, Page No: 102
Obstetric Significance of Plane of Least Pelvic Dimension
▪ It is the narrowest plane in the pelvis
▪ This plane corresponds roughly to the origin of levator ani muscles
▪ It is at this plane that the internal rotation of the fetal head occurs
▪ It marks the beginning of the forward curve of the pelvic axis
▪ It is a landmark used for pudendal nerve block analgesia
▪ This level of ischial spines indicates Station ‘O’
▪ It is irregularly oval and notched on each side by ischial spine.

52. A female has strange feeling during Puerperium. She is anxious, tearful, insomniac and at times helpless.
Treatment of such condition is:
a. Estrogen supplement

Paradise Institute & Self Study Centre CALL/VIBER: 9818723799 Page 22


PARADISE WEEKLY MODEL TEST – CEE MD/MS BASED POUSH 09

b. Reassurance
c. SSRI
d. ECT

Ans: b
Ref: Dutta Obstetrics 8th Edition, Page No: 512
Puerperal blues
▪ It is a transient state of mental illness observed 4–5 days after delivery and it lasts for a few days.
▪ Nearly 50% of the postpartum women suffer from the problem.
▪ Manifestations are—depression, anxiety, tearfulness, insomnia, helplessness and negative feelings toward
the infant.
▪ No specific metabolic or endocrine abnormalities have been detected. But lowered tryptophan level is
observed. Itsuggests altered neurotransmitter function.
▪ Treatment is reassurance and psychological support by the family members

53. Cervical incompetency during pregnancy is indicated by:


a. Painless cervical shortening and dilatation
b. Short cervix < 25 mm in sonography
c. Detection of dilatation of internal os with herniation of the membranes
d. All of the above

Ans: d
Ref: DC Dutta Textbook of Obstetrics including Perinatology and Contraception 8th Edition, Page No: 198, 201
Cervical incompetency during Pregnancy:
▪ Clinical (speculum): Painless cervical shortening and dilatation.
▪ Sonography: Short cervix < 25 mm; Funneling of the internal os > 1 cm.
▪ Speculum examination: Detection of dilatation of internal os with herniation of the membranes.
Cervical incompetence is primarily a clinical diagnosis. It is manifested with recurrent painless cervical dilatation and
spontaneous midtrimester miscarriage. USG diagnosis (during pregnancy) : Short cervix <25 mm length and funneling of the
internal Os >1 cm.
Cervical incompetence is the ultimate manifestation of cervical weakness. It may either be due to the primary anatomic
defect of the cervix or due to trauma or may be due to the complex pathology of infection or inflammation of
decidua/membranes leading to uterine contractions and cervical ripening.

54. Twin peak sign is mostly suggestive of:


a. Dichorionicity
b. Monochorionicity
c. Diamniocity
d. Monoamniocity

Ans: a
Ref: Twin-peak sign (twin pregnancy) by Craig Hacking et al in Radiopedia; DC Dutta Textbook of Obstetrics
including Perinatology and Contraception 8th Edition, Page No: 236
▪ The twin peak sign (also known as the lambda (λ) sign) is a triangular appearance of the chorion insinuating between
the layers of the intertwin membrane and strongly suggests a dichorionic twin pregnancy.
▪ It is best seen in the first trimester (between 10-14 weeks).
▪ While the presence of a twin peak sign is a useful indicator of dichorionicity, its absence, however, is not that useful
in confidently excluding it.

55. A primipara is in the second stage of labor and an episiotomy is about to be cut. Compared with a midline
episiotomy, which of the following is an advantage of mediolateral episiotomy?

Paradise Institute & Self Study Centre CALL/VIBER: 9818723799 Page 23


PARADISE WEEKLY MODEL TEST – CEE MD/MS BASED POUSH 09

a. Ease of repair
b. Less blood loss
c. Less dyspareunia
d. Less extension of the incision

Ans: d
Ref: Cunningham, p 401-402.
Midline episiotomies are easier to fix and have a smaller incidence of surgical breakdown, less pain, and lower blood loss.
The incidence of dyspareunia is somewhat less. However, the incidence of extensions of the incision to include the rectum is
considerably higher than with mediolateral episiotomies. Regardless of technique, attention to hemostasis and anatomic
restoration is the key element of a technically appropriate repair.

56. With oral iron therapy, rise in Hb% can be seen after:
a. 3 weeks
b. 4 weeks
c. 6 weeks
d. 8 weeks

Ans: a
Ref: Dutta Obstetrics 8th Edition, Page No: 309
Early detection of falling hemoglobin level is to be made. Hemoglobin level should be estimated at the first antenatal
visit, at the 30th week and finally at 36th week.
Response of iron therapy is evidenced by:
▪ Sense of well-being
▪ Increased appetite
▪ Improved outlook of the patient
▪ Hematological examination: (a) Rise in hemoglobin level, (b) hematocrit value returning to normal, (c)
reticulocytosis within 7–10 days.
If no significant improvement is evident clinically and hematologically within 3 weeks, diagnostic re-evaluation is needed.
Rise in hemoglobin with oral iron – 0.7 gm-1 gm per week, which is seen after 3 weeks of initiation of oral therapy.
Hemoglobin levels should increase by at least 0.3g/dl/week if the patient is responding to therapy.
If there is no significant clinical or hematological improvement within 3 weeks, diagnostic re-evaluation is needed.

57. A 24-year-old primigravida presents for routine ultrasound at 20 weeks gestation. Based on the ultrasound
findings, the patient is diagnosed with twin boys. Which of the following is true statement regarding the
membranes and placentas of this patient’s twins if they are dizygotic?
a. They are dichorionic and monoamniotic because the fetuses are of the same sex.
b. They must be monochorionic and monoamniotic because they are of the same sex.
c. They cannot be monochorionic and monoamniotic.
d. They cannot be dichorionic and diamniotic.

Ans: c
Ref: Cunningham, pp 859-865
• Dizygotic twins cannot be monochorionic and monoamnionic since they are the result of fertilization of two eggs.
Therefore they always have a dichorionic and diamniotic placenta regardless of the sex of the fetuses.
• The placentas of dizygotic twins may be totally separate or intimately fused depending upon the location of
implantation of the two zygotes.
• Monozygotic twins are always of the same sex since they derive from the division of one zygote but may be
monochorionic or dichorionic depending upon when the separation of the twins occurred. Of monozygotic twins,
20% to 30% have dichorionic placentation, the result of separation of the blastocyst in the first 2 days after
fertilization. The majority of monozygotic twins have a diamniotic and monochorionic placenta. The least common

Paradise Institute & Self Study Centre CALL/VIBER: 9818723799 Page 24


PARADISE WEEKLY MODEL TEST – CEE MD/MS BASED POUSH 09

type of placentation in monozygotic twins is the monochorionic and monoamniotic placenta; its incidence is only
about 1%. Conjoined twins are always monozygotic.

58. Forceps should not be used in:


a. Hydrocephalus
b. Severe preeclampsia
c. Post maturity
d. After coming head

Ans: a
Ref: DC Dutta Textbook of Obstetrics including Perinatology and Contraception 8th Edition, Page No: 654
Indications for Operative Vaginal Delivery (Forceps/Ventouse):

59. The following are related to fetal erythropoiesis except:


a. In the embryonic phase, the erythropoiesis is first demonstrated in the primitive mesoderm
b. By 10th week, the liver becomes the major site
c. Near term, the bone marrow becomes the major site
d. At terms 75–80% of hemoglobin is fetal type (HbF)

Ans: a
Ref: Dutta Obstetrics 8th Edition, Page No: 47

• Hematopoiesis is demonstrated in the embryonic phase first in the yolk sac by 2-3 weeks.
• During the first half, the hemoglobin is of fetal type (α-2, g-2) but from 24 weeks onwards, adult type of
hemoglobin (α-2, β-2) appears and at term about 75–80% of the total hemoglobin is of fetal type HbF. Between
6– 12 months after birth, the fetal hemoglobin is completely replaced by adult hemoglobin.
• Difference between HbA and HbF is that fetal hemoglobin has got a greater affinity to oxygen due to lower
binding of 2, 3-diphosphoglycerate compared to adult hemoglobin. It is also resistant to acid and alkali.

60. All of the following appear to decrease hot flushes in menopausal women except:
a. Androgens
b. Raloxifene
c. Isoflavones
d. Tibolone

Paradise Institute & Self Study Centre CALL/VIBER: 9818723799 Page 25


PARADISE WEEKLY MODEL TEST – CEE MD/MS BASED POUSH 09

Ans: b
Ref: DC Dutta Textbook of Gynecology including Contraception; 6th Edition, Page No: 61
Raloxifene is an orally administered selective estrogen receptor modulator (SERM). It has estrogenic action on bone and anti-
estrogenic actions on the uterus and breast. It is used to prevent osteoporosis postmenopausal women. It is also protective
against breast and endometrial cancer. It is not useful in treatment of hot flushes.

Orthopedics
61. A 49-year-old “tweaks” his neck while loading heavy equipment atop his family SUV. He complains of 3 days of
small finger numbness and difficulty gripping. Which of the following nerve roots is most likely entrapped?
a. C6
b. C7
c. C8
d. T1

Ans: c
Ref: Rhee JM, Yoon T, Riew D. Cervical radiculopathy. J Am Acad Orthop Surg. 2007; 15:486-494.
Grip strength and sensation to the small and sometimes ring fingers are classically attributed to the C8 nerve root, most often
affected by C7-T1 disc herniations.

62. What is not an absolute indication of open reduction?


a. Displaced intra-articular fracture.
b. Type IV Displaced epiphyseal injuries
c. Nonunion
d. Multiple fractures

Ans: d
Ref: Maheshwari Orthopedics, 5th edition, page 16
• Indications for open reduction
• Absolute
o Failure of closed reduction
o Displaced intra-articular fractures.
o Some displaced epiphyseal injuries (types III and IV)

Paradise Institute & Self Study Centre CALL/VIBER: 9818723799 Page 26


PARADISE WEEKLY MODEL TEST – CEE MD/MS BASED POUSH 09

o Major avulsion fractures e.g., fracture of patella


o Non-union
• Relative
o Delayed union and malunion
o Multiple fractures
o Pathological fractures
o Where closed reduction is known to be ineffective e.g., fracture of the neck of the femur
o Fractures with vascular or neural injuries

63. Which of the following does ASIA impairment scale B represent?


a. Normal motor and sensory function
b. Motor function is preserved below the neurological level, and more than half of key muscles below the
neurological level have a muscle grade less than 3
c. No motor or sensory function is preserved in the sacral segments S4-S5
d. Sensory but not motor function is preserved below the neurological level and includes the sacral segments S4-S5

Ans: d
Ref: Apley & Solomon’s System of Orthopedics & Trauma, 10th edition, page 839

64. A 44-year-old woman suffers a fall while hiking and landing on her buttocks and falling forward. She complains
of bilateral pain in her legs, distal to her knees. She has profound weakness in her bilaterally extensor hallucis
longi and gastrosoleus complexes and has marked saddle anesthesia. MRI was done and shows a large, midline
herniated disc, compressing each of the traversing nerve roots and entire cauda equina below its level, but sparing
the exiting nerve roots. Which disc is most likely involved in this injury?
a. L3-4
b. L4-5
c. L5-S1
d. S1-2

Ans: b
Ref: Maheshwari Orthopedics, 5th edition, page 17
This patient has weak extensor hallucis longi (L5 nerve root), weak gastrocsoleus complexes (S1), and saddle anesthesia (S2-
4). She has normal function above this level, suggesting that the L5 nerve roots and those below are affected. An injury at L4-
5 that spares the exiting roots (L4) but affects the traversing roots (L5) and those below (S1-5) would be most consistent with
these symptoms. Remember the sacral spine does not have interbody discs, as it is fused.

Paradise Institute & Self Study Centre CALL/VIBER: 9818723799 Page 27


PARADISE WEEKLY MODEL TEST – CEE MD/MS BASED POUSH 09

Paradise Institute & Self Study Centre CALL/VIBER: 9818723799 Page 28


PARADISE WEEKLY MODEL TEST – CEE MD/MS BASED POUSH 09

65. A 44-year-old cab driver is involved in a high-speed, head-on collision. In the ED, he is hemodynamically stable
and has full neurologic function of his lower extremities. He is complaining of severe lower back pain.
Radiographs and CT imaging are acquired of his lumbar spine. Which of the following findings is indicative of an
unstable lumbar burst fracture that may need surgical stabilization?
a. 50% loss of vertebral height at the level of injury
b. 20 degrees of kyphosis at the level of injury
c. Injury to the anterior and middle spinal columns only
d. Injury to the posterior column only

Ans: A
Ref: Egol KE, Koval KJ, Zukerman JD, eds. Thoracolumbar spine. In: Handbook of Fractures. 4th ed. Philadelphia:
Lippincott Williams & Wilkins; 2010:123-140.
Lumbar burst fractures are considered unstable and indicate surgical stabilization if they involve all 3 columns, result in >
40% to 50% loss of height or > 30 degrees of kyphosis, or have > 40% canal compromise. Burst fractures by definition are
not exclusive to the posterior elements.

66. Anconeus triangle formed by all of the following except?


a. Lateral epicondyle
b. Radial head
c. Tip of the olecranon
d. Ulnar head

Ans: D
Ref: Ref: Maheshwari Orthopedics, 5th edition, page 107
Anconeus triangle formed by radial head, lateral epicondyle and the tip of the olecranon

67. What is Spoon plate used for?


a. Fracture of forearm bones
b. Lower end of tibia fracture
c. Hip arthrodesis.
d. Fracture of shaft of femur

Ans: b
Ref: Ref: Maheshwari Orthopedics, 5th edition, page 19

Paradise Institute & Self Study Centre CALL/VIBER: 9818723799 Page 29


PARADISE WEEKLY MODEL TEST – CEE MD/MS BASED POUSH 09

68. In what position are fractures of both bones of the forearm, above the insertion of the pronator teres is
immobilized?
a. Mid prone
b. Pronation
c. Semi supine
d. Supination

Ans: d
Ref: Maheshwari Orthopedics, 5th edition, page 116
Fractures of both bones of the forearm, above the insertion of the pronator teres is immobilized in supination, below the
insertion of the pronator teres is immobilised in mid-neutral position.

69. A 15-year-old girl has asymptomatic adolescent idiopathic scoliosis. Her parents ask you how debilitated she
might be compared with her peers who do not have scoliosis. Which of the following is true regarding the natural
history of idiopathic scoliosis?
a. Difficulty with pregnancy
b. Increased risk of developing cancer
c. Acute or chronic back pain
d. Athletic limitations

Paradise Institute & Self Study Centre CALL/VIBER: 9818723799 Page 30


PARADISE WEEKLY MODEL TEST – CEE MD/MS BASED POUSH 09

Ans: c
Ref: Apley & Solomon’s System of Orthopaedics & Trauma, 10 th edition
Scoliotic patients have been found to have more acute or chronic back pain, as well as cosmetic concerns. Pregnancy
complications, athletic limita- tions, and an increased cancer risk are not associated with AIS.

70. What is not a feature of inter-trochanteric fracture?


a. Associated with significant trauma
b. Ecchymosis present
c. Tenderness over the greater trochanter
d. Walking may be possible in impacted fracture

Ans: D
Ref: Maheshwari Orthopedics, 5th edition, page 134

ENT

71. What is an indication of intranasal inferior meatal antrostomy?


a. Chronic purulent maxillary sinusitis
b. Osteitis
c. Polypoidal hypertrophy
d. Suspicious of malignancy

Ans: A
Ref: Dhingra ENT, 7th edition, page 467
Intranasal inferior meatal antrostomy is a process of making an opening in the nasoantral wall of the inferior meatus
by intranasal route. This operation is now rarely required and has been superseded by functional endoscopic sinus
surgery.
Indications
o Chronic purulent maxillary sinusitis.
Contraindications
o Irreversible change in sinus mucosa, e.g. polypoidal hypertrophy.
o Presence of osteitis.
o Suspicion of malignancy.

72. What is the key indicator of herpes zoster oticus (Ramsay Hunt syndrome)?
a. Vesicles on an erythematous base found in the external ear
b. Noncaseating granulomas on lower lip biopsy
c. Circulating antibodies to Borrelia burgdorferi

Paradise Institute & Self Study Centre CALL/VIBER: 9818723799 Page 31


PARADISE WEEKLY MODEL TEST – CEE MD/MS BASED POUSH 09

d. Loss of taste on the ipsilateral tongue

Ans: a
Ref: Dhingra ENT, 7th edition, Current diagnosis and treatment. Cleve Clin J Med 2005;72(5):398–401, 5.
The pathognomonic feature of herpes zoster oticus (Ramsay Hunt syndrome) is a vesicular eruption on an erythematous base
in an area of facial nerve sensory distribution (external ear). This disorder is caused by reactivation of varicella-zoster virus
and is treated with antiviral medications and steroids. Inadequately treated zoster infections can lead to poor recovery of facial
function and postherpetic neuralgia.

73. What is not a feature of Rhinosporidiosis?


a. Chronic granulomatous disease of nose
b. Lesion is studded with white dots
c. Russell bodies
d. Upon touching bleeding occurs

Ans: C
Ref: Ref: Dhingra ENT, 7th edition, page 175 – 178
o Russell bodies are seen in biopsy of Rhinoscleroma.
o Rhinosporidiosis is a chronic granulomatous disease caused by Rhinosporidium seeberi and affects both man and
animals.
o The disease mostly affects nose and nasopharynx; other sites such as lip, palate, conjunctiva, epiglottis, larynx,
trachea, bronchi, skin, vulva and vagina may also be affected.
o The disease is acquired through contaminated water of ponds also frequented by animals. In the nose, the disease
presents as a leafy, polypoidal mass, pink to purple in colour and attached to nasal septum or lateral wall. Sometimes,
it extends into the nasopharynx and may hang behind the soft palate. The mass is very vascular and bleeds easily on
touch. Its surface is studded with white dots representing the sporangia. In early stages, the patient may complain of
nasal discharge which is often blood tinged and nasal stuffiness. Sometimes, frank epistaxis is the only presenting
complaint.

74. What is Chevallet fracture of nasal septum?


a. Comminuted
b. Horizontal
c. Serpentine
d. Vertical

Ans: d
Ref: Dhingra ENT, 7th edition, page 165
“Jarjaway” fracture of nasal septum results from blows from the front; it starts just above the anterior nasal spine
and runs horizontally backwards just above the junction of septal cartilage with the vomer.
“Chevallet” fracture of septal cartilage results from blows from below; it runs vertically from the anterior nasal spine
upwards to the junction of bony and cartilaginous dorsum of nose.

75. Where does Schneiderian papilloma arise from?


a. Lateral wall of nasal cavity
b. Medial wall of nasal cavity
c. Paranasal sinuses
d. Posterior pharyngeal space

Ans: A
Ref: Dhingra ENT, 7th edition, page 227

Paradise Institute & Self Study Centre CALL/VIBER: 9818723799 Page 32


PARADISE WEEKLY MODEL TEST – CEE MD/MS BASED POUSH 09

▪ Inverted Papilloma (Transitional Cell Papilloma or Ringertz Tumour or Schneiderian Papilloma) is a tumour of the
nonolfactory mucosa of nose (Schneiderian membrane) and paranasal sinuses. Most common site of origin is lateral
wall of nose in the middle meatus; less commonly it arises from the maxillary, frontal or sphenoid sinus.
▪ It is so named because hyperplastic papillomatous tissue grows into the stroma rather than in exophytic manner.
Human papilloma virus is thought to be responsible for its aetiology. Clinically, men are affected more than women
in the age group of 40–70. It is almost always unilateral and presents with nasal obstruction, nasal discharge and
epistaxis. It can invade sinuses or orbit. Orbital involvement causes proptosis, diplopia and lacrimation. On
examination of nose or endoscopy, it presents as a pale polypoidal mass resembling a simple nasal polypus or polypi.
▪ Computed tomography (CT) and magnetic resonance imaging (MRI) show the location and extent of the lesion. MRI
also helps to differentiate associated secretions in sinus from the actual tumour mass. Biopsy is essential for diagnosis.
Care should be taken as simple nasal polypi may be associated with it or even the patient might have been operated
for their removal.
▪ Treatment. Medial maxillectomy is the treatment of choice. It can be performed by lateral rhinotomy or sublabial
degloving approach. These days endoscopic approach is preferred. In 10–15% of cases, it is associated with
malignancy. Wider external surgical approaches may be required for tumour extending to the frontal sinus or orbit.
Recurrence can occur. Radiotherapy is not advised as it may induce malignancy.

76. You have been asked to see a patient with a low calcium level. She has recently had a total thyroidectomy. Which
of the following signs would you look for to suggest hypocalcemia, EXCEPT?
a. Trousseau sign.
b. Chvostek’s sign.
c. De Musset’s sign.
d. Carpopedal spasm.

Ans: c
Ref: Dhingra ENT, 7th edition,
• One of the complications of a thyroidectomy is damage to the parathyroid glands. These glands are intimately linked
to the thyroid gland and are located just behind it. The parathyroid glands are responsible for calcium homeostasis
within the body. When removing the thyroid gland it is possible to accidently damage or remove the parathyroid
glands as well. As a result, the patient may become hypokalemic if they are not given calcium supplementation. The
Trousseau sign is elicited by inflating a blood pressure cuff, which results in carpal spasm in hypocalcemia.
• Chvostek’s sign is elicited by tapping in the region of the facial nerve, which results in a facial spasm in hypocalcemia.
• De Musset’s sign is the rhythmic head bobbing that is seen in aortic regurgitation.
• Carpopedal spasm: This characteristic posture is seen in hypocalcemia.

77. What is Wigand’s technique of endoscopic sinus surgery?


a. Anterior to posterior
b. Medial to anterior
c. Medial to posterior
d. Posterior to anterior

Ans: d
Ref: Dhingra ENT, 7th edition, page 477
• Two surgical techniques of endoscopic sinus surgery are followed:
• Anterior to posterior (Stammberger’s technique). In this technique surgery proceeds from uncinate process backward
to sphenoid sinus. Advantage of this technique is to tailor the extent of surgery to the extent of disease.
• Posterior to anterior (Wigand’s technique). Surgery starts at the sphenoid sinus and proceeds anteriorly along the base
of skull and medial orbital wall. This is mostly done in extensive polyposis or in revisional sinus surgery.

78. What is an early complication of cochlear implant surgery?


a. CSF leak

Paradise Institute & Self Study Centre CALL/VIBER: 9818723799 Page 33


PARADISE WEEKLY MODEL TEST – CEE MD/MS BASED POUSH 09

b. Exposure of device and extrusion


c. Pain at site of implant
d. Otitis media

Ans: a
Ref: Dhingra ENT, 7th edition, page 141

79. What is Lyre sign suggestive of?


a. Carotid body tumor
b. Grave’s disease
c. Lateral sinus thrombosis
d. Meniere’s disease

Ans: A
Ref: Dhingra ENT, 7th edition, page 447
▪ Carotid body tumour arises from the chemoreceptor cells in the carotid body, hence also called chemodectoma. Mostly
presents after 40 years. It is a very slow-growing tumour and the history of mass in the neck may extend into several
years. It presents as a painless swelling which is pulsatile. Bruit can be heard with a stethoscope. It moves from side
to side but not vertically. It may extend into the parapharyngeal space and present in the oropharynx.
▪ Contrast-enhanced CT and MRI with gadolinium are diagnostic and also show the extent of the tumour. MRI
angiography shows splaying of internal and external carotid arteries (Lyre’s sign). Some tumours are functional and
secrete catecholamines. Hence serum catecholamines and urinary metanephrines and vanillylmandelic acid (VMA)
should be estimated. Fine-needle aspiration cytology (FNAC) or biopsy should not be done because of the vascularity
of tumour.
▪ Treatment is surgical when the patient is younger than 50 years and surgically fit, or when the tumour extends into
the oropharynx causing difficulty in speech, swallowing or breathing. Radiotherapy is also effective and is used in
older patients and those unfit for surgery or those who refuse surgery or have a metastatic disease.

80. What is the average hearing loss due to complete obstruction of ear canal?
a. 30 dB
b. 40 dB
c. 60 dB
d. 90 dB
Ans : a
Ref: ENT Dhingra, 7th edition, page 31
o Average hearing loss seen in different lesions of conductive apparatus
o Complete obstruction of ear canal: 30 dB
o Perforation of tympanic membrane (It varies and is directly proportional to the size of perforation):

Paradise Institute & Self Study Centre CALL/VIBER: 9818723799 Page 34


PARADISE WEEKLY MODEL TEST – CEE MD/MS BASED POUSH 09

10–40 dB
o Ossicular interruption with intact drum: 54 dB
o Ossicular interruption with perforation: 38 dB
o Malleus fixation: 10–25 dB
o Closure of oval window: 60 dB

Ophthalmology
81. A 50-year-old woman is diagnosed with acute angle-closure glaucoma. She has acutely decreased visual acuity.
Which of the following is the most likely mechanism for this condition?
a. Increased IOP caused by increased aqueous humor production
b. Nonreactive pupil leading to increased intraocular pressure
c. Decreased outflow of the aqueous humor
d. Separation of the retina leading to decreased visual acuity

Ans: C
Ref: Khurana Ophthalmology, 6th edition
In acute angle-closure glaucoma the sudden rise in IOP is a consequence of blocked outflow, usually due to a pupillary block,
and not increased production of aqueous humor.

82. Which of the following is a cause of passive edema of the eyelids?


a. Acute Dacrocystitis
b. Dermatitis
c. Maxillary sinusitis
d. Severe anemia

Ans: D
Ref: Khurana Ophthalmology, 6th edition, page 365
o Owing to the looseness of the tissues, oedema of the lids is of common occurrence. It may be classified as
inflammatory, solid and passive oedema.
o Inflammatory oedema. It is seen in the following conditions.
o Inflammations of the lid itself, which include dermatitis, stye, hordeolum internum, insect bites, cellulitis and
lid abscess.
o Inflammations of the conjunctiva, such as acute purulent, membranous and seudomembranous conjunctivitis.
o Inflammations of the lacrimal sac, i.e., acute dacryocystitis and lacrimal abscess.
o Inflammations of the lacrimal gland, i.e., acute dacryoadenitis.
o Inflammations of the eyeball, such as acute iridocyclitis, endophthalmitis and panophthalmitis.
o Inflammations of the orbit, which include orbital cellulitis, orbital abscess and pseudotumour.
o Inflammations of the paranasal sinuses, e.g., maxillary sinusitis.
o Solid oedema of the lids. It is chronic thickening of the lids, which usually follows recurrent attacks of erysipelas. It
resembles inflammatory oedema of the lids but is harder in consistency.
o Passive oedema of the lids. It may occur due to local or general causes.
o Local causes are: cavernous sinus thrombosis, head injury and angioneurotic oedema.
o General causes are congestive heart failure, renal failure, hypoproteinaemia and severe anaemia.

83. Trachoma does not end up causing which of the following eye changes?
a. Corneal pannus
b. Ectropion
c. Madarosis
d. Tylosis

Ans: B
Ref: Khurana Ophthalmology, 6th edition, page 69 – 70
The end stage of cicatricial trachoma is also referred to as sequelae of trachoma. They are as follows:

Paradise Institute & Self Study Centre CALL/VIBER: 9818723799 Page 35


PARADISE WEEKLY MODEL TEST – CEE MD/MS BASED POUSH 09

▪ Conjunctival signs: Conjunctival scarring, concretions, pseudocyst, xerosis and symblepharon.


▪ Corneal signs: Regressive pannus, Herbert pits, corneal opacity, corneal ectasia, corneal xeerosis and total corneal
pannus.
▪ Lid signs: Trichiasis, entropion, tylosis, ptosis, madarosis and anklyoblepharon.
▪ Lacrimal apparatus sequelae: Chronic dacrocystitis and chronic dacroadenitits.

84. Which of the following is responsible for producing upward and outward movement of the eye ball?
a. Inferior oblique
b. Lateral rectus
c. Medial rectus
d. Superior oblique

Ans: a
Ref: Khurana Ophthalmology, 6th edition, page 561
Muscle Primary action Secondary action Tertiary action
Medial rectus Adduction -
Lateral rectus Abduction -
Superior rectus Elevation Intorsion Adduction
Inferior rectus Depression Extorsion Adduction
Superior oblique intorsion Depression Abduction
Inferior oblique Extorsion Elevation Abduction

85. What is a feature of deep corneal vascularization?


a. Corneal surface is not disturbed
b. Well defined bright vessels seen
c. Vessels branch in an arborescent fashion
d. Vessels end traced over the limbus into the conjunctiva

Ans: a
Ref: Khurana Ophthalmology, 6th edition, page 503
Differences between superficial and deep corneal vascularization
Superficial corneal vascularization Deep corneal vascularization
Corneal vessels can be traced over the limbus into the Corneal vessels abruptly end at the limbus.
conjunctiva.
Vessels are bright red and well-defined. Vessels are III-defined and cause only a diffuse reddish blush.
Superficial vessels branch in an arborescent manner. Deep vessels run parallel to each other in a radial fashion.
Superficial vessels raise the epithelium and make the Deep vessels do not disturb the corneal surface.
corneal surface irregular

86. What is used as a mainstay for management of Eales disease with active retinal vasculitis?
a. Anti-tubercular therapy
b. Laser photocoagulation
c. Oral corticosteroids
d. Vitreous surgery

Ans: C
Ref: Khurana Ophthalmology, 6th edition, page 268 – 269

Paradise Institute & Self Study Centre CALL/VIBER: 9818723799 Page 36


PARADISE WEEKLY MODEL TEST – CEE MD/MS BASED POUSH 09

Eales disease is an idiopathic inflammation of the peripheral retinal veins. It is characterized by recurrent vitreous
haemorrhage; so also referred to as primary vitreous haemorrhage. The disease is rare in Caucasians but is an important cause
of visual morbidity in young Asian males.
Treatment of Eales’ disease comprises:
▪ Medical treatment. Course of oral corticosteroids for extended periods is the mainstay of treatment during stage of
active inflammation. A course of antitubercular therapy has also been recommended in selective cases.
▪ Laser photocoagulation of the retina either PRP or feeder vessel photocoagulation is indicated in stage of
neovascularizion.
▪ Vitreoretinal surgery is required for non-resolving vitreous haemorrhage and fractional retinal detachment.

87. What is retinoblastoma associated with?


a. De Grouchy syndrome
b. Edwards syndrome
c. Cri-du-Chat syndrome
d. Patau syndrome

Ans: c
Ref: Khurana Ophthalmology, 6th edition, page 473
Cri-du-Chat syndrome (13q.)
• Retinoblastoma
• Hypertelorism
• Microphthalmos
• Epicanthus
• Ptosis
• Coloboma
• Cataract

Anesthesiology
88. What is the most rapid method to decrease the intra cranial tension in a patient in ICU awaiting surgery?
a. Dexamethasone
b. Frusemide
c. Hyperventilation
d. Mannitol

Ans: C
Ref: Ajay Yadav Anesthesia, 6th edition, page 223
• Hyperventilation It is the most rapid method of reducing ICT.
• Hyperventilation reduces the ICT by causing hypocapnia which in turn causes cerebral vasoconstriction to decrease
the production of cerebrospinal fluid (CSF).
• Moderate hypocapnia is beneficial however excessive hypocapnia by causing excessive cerebral vasoconstriction can
cause cerebral ischemia therefore it is recommended to maintain pCO2 between 25-30 mm Hg.

89. W For any anesthetics, the basic standards of monitoring include a continuous evaluation of all but which one of
the following?
a. Oxygenation
b. Ventilation
c. Hematocrit
d. Temperature

Ans: c
Ref: Ajay Yadav Anesthesia, 6th edition

Paradise Institute & Self Study Centre CALL/VIBER: 9818723799 Page 37


PARADISE WEEKLY MODEL TEST – CEE MD/MS BASED POUSH 09

The basic monitoring standards of the American Society of Anesthesiologists states that “the patient’s oxygenation,
ventilation, circulation, and temperature shall be continually evaluated.” D, continuous monitoring of the hematocrit, is not
necessary and even during cases associated with a massive blood loss, is rarely accomplished.

90. Adrenocortical suppression is seen with which induction agent?


a. Propofol
b. Ketamine
c. Etomidate
d. Thiopentone

Ans: C
Ref: Miller’s Anesthesia, 8th edition, page 850 – 851
• Etomidate was introduced into clinical practice in 1972. The unique properties of etomidate include hemodynamic
stability, minimal respiratory depression, cerebral protection, favorable toxicity profile, and pharmacokinetics
enabling rapid recovery after either a single dose or a continuous infusion.
• In the 1970s, these beneficial properties led to widespread use of etomidate for induction, for maintenance of
anesthesia, and for prolonged sedation in critically ill patients. The enthusiasm among clinicians for etomidate was
tempered in the 1980s by reports that the drug can cause temporary inhibition of steroid synthesis after single doses
and infusions. This effect, combined with other minor disadvantages (e.g., pain on injection, superficial
thrombophlebitis, myoclonus, and a frequent incidence of nausea and vomiting), led to several editorials questioning
the role of etomidate in modern anesthetic practice.
• Use of the drug decreased after those editorials, but it has expanded again as a result of the rediscovery of etomidate’s
beneficial physiologic profile and a widening use in emergency departments and ICUs, combined with a lack of novel
reports describing clinically significant adrenocortical suppression after induction or brief infusions.

91. A During a rapid sequence induction, when would it be appropriate to release cricoid pressure?
a. When the patient becomes unconscious.
b. After the induction agent has been given but before the neuromuscular blocking drug is given.
c. After proper placement of the endotracheal tube has been confirmed.
d. Cricoid pressure is not indicated in this instance.

Ans: C
Ref: Ajay Yadav Anesthesia, 6th edition
Cricoid pressure or Sellick’s maneuver is intended to occlude the esophagus to prevent passive regurgitation of gastric
contents into the airway. It should be maintained throughout the induction sequence until proper endotracheal tube placement
has been con- firmed or until successful intubation occurs and during mask ventila- tion, if needed. Cricoid pressure is
indicated in this case.

92. A 35-year-old G4P2 woman at 37 EGA was diagnosed with severe PIH. She is treated with magnesium infusion
and labor induction is started. She is noted to have acute mental status changes, followed by a seizure, and an
abrupt decrease in her O2 saturation to 85%. Which of the following is the most likely reason for her hypoxia?
a. Pulmonary edema
b. Upper airway obstruction
c. Pneumothorax
d. Pulmonary aspiration

Ans: B
Ref: Morgan Anesthesia, 4th edition
Given the abrupt reduction in her SaO2, the most likely diagnosis in this postictal patient is upper airway obstruction, which
should be treated with an elevation of the mandible while oxygen is administered. Airway edema, pulmonary edema, and
aspiration are all also possible causes of hypoxemia associated with eclampsia. However, the sudden desaturation makes them
less likely. Spontaneous pneumothorax is much less likely in this patient.

Paradise Institute & Self Study Centre CALL/VIBER: 9818723799 Page 38


PARADISE WEEKLY MODEL TEST – CEE MD/MS BASED POUSH 09

93. A 2-year-old boy is having correction of ptosis of the right eyelid. The patient is stable during induction of
anesthesia, but his heart rate drops to 50 bpm soon after the surgical incision. Which of the following is your first
step in treating this patient?
a. Do nothing; his heart rate is in the normal range.
b. Give intravenous atropine.
c. Tell the surgeon to stop what he is doing.
d. Give intravenous fluids.

Ans: C
Ref: Ajay Yadav Anesthesia, 6th edition
The patient is having a vagal response to something the surgeon is doing on or around his eye. The first step in treating his
bradycardia is to have the surgeon stop until the patient’s heart rate returns to normal (110-115 bpm). If the pause in surgery
does not rapidly return the heart rate to normal, intravenous atropine should be given.

94. What is the choice of anesthesia for caesarean section in patients with pregnancy induced hypertension?
a. Epidural
b. General
c. Spinal
d. Total intravenous

Ans: A
Ref: Ajay Yadav Anesthesia, 6th edition, page 233
▪ Anesthetic Considerations for Cesarean Section in Patients with Pregnancy induced Hypertension/ Preeclampsia
▪ Before giving central neuraxial blocks it is important to rule out HELLP syndrome.
▪ Anesthesia of Choice: Anesthetic technique of choice is epidural. Difficult intubation due to laryngeal edema can
make GA to become risky. As PIH patients are hypertensive they are more prone for hypotension however, if
necessary, spinal can be given to PIH patients.
▪ If the patient is having HELLP syndrome then epidural also becomes contraindicated and patient receives GA.

Psychiatry
95. Which substance of abuse produces most psychic dependence?
a. Cocaine
b. Phencyclidine
c. LSD
d. Opioids

Ans: D
Ref: Ahuja Psychiatry, 7th edition, page 35

Paradise Institute & Self Study Centre CALL/VIBER: 9818723799 Page 39


PARADISE WEEKLY MODEL TEST – CEE MD/MS BASED POUSH 09

96. What is a person with apathy, lack of initiative and retardation is likely suffering from?
a. Frontal convexity syndrome
b. Medial frontal syndrome
c. Orbito-frontal syndrome
d. Temporal lobe syndrome

Ans: a
Ref: Ahuja Psychiatry, 7th edition, page 31

97. The observation that levodopa (a drug used to treat Parkinson disease) can cause mania and psychosis in some
patients supports which neurochemical theory of psychiatric behavior?
a. Norepinephrine

Paradise Institute & Self Study Centre CALL/VIBER: 9818723799 Page 40


PARADISE WEEKLY MODEL TEST – CEE MD/MS BASED POUSH 09

b. Dopamine
c. Glycine
d. Glutamine

Ans: b
Ref: (Kaplan and Sadock, p 303.)
Levodopa is a chemical relative of dopamine. The fact that a dopamine-related compound can cause psychotic symptoms
in some patients supports the dopamine hypothesis of schizophrenia, which is the leading neurochemical hypothesis for
this disease.

98. Benzodiazepines, barbiturates, and many anticonvulsants exert their influence through which of the following
types of receptors?
a. Muscarinic
b. Dopaminergic
c. Adrenergic
d. γ-Aminobutyric acid (GABA)–ergic

Ans: D
Ref: (Kaplan and Sadock, pp 948-953.)
GABA receptors represent the most important inhibitory system in the central nervous system (CNS) and are found in almost
every area of the brain. Benzodiazepines, barbiturates, and many anticonvulsants act through activation of the GABA
receptors. This explains the cross-tolerance that occurs between these substances

99. A 46-year-old man is being monitored in a sleep study laboratory. After he has been asleep for 90 minutes, his
EEG shows low-voltage, random fast activity with sawtooth waves. When awakened during this period, the patient
reports that he was dreaming. Which of the following sleep stages was this patient in when awakened?
a. Alpha waves
b. Theta waves
c. Delta waves
d. Rapid eye movement (REM)

Ans: d
Ref: (Kaplan and Sadock, p 535.)
Dreaming is the main characteristic of REM sleep. The EEG shows characteristic low-voltage waves that are random, fast,
and sawtoothed. Active eye movements are attributed to the individual’s “watching” his or her dreams. A lack of muscle tone
during REM sleep prevents the individual from acting out his or her dreams. REM sleep is also characterized by increased
heart rate and blood pressure and penile or clitoral nocturnal erections.

100. A 38-year-old man comes to his physician with complaints of impaired ejaculation. He is on the following
medications: perphenazine, digoxin, and propranolol. He is also receiving methadone treatment and admits to
periodic cannabis use. Which substance is the most likely culprit in his problems with ejaculation?
a. Perphenazine
b. Digoxin
c. Propranolol
d. Cannabis

Ans: A
Ref: (Kaplan and Sadock, p 584.)
Perphenazine is known to cause impairment in ejaculation. The other drugs in the option list can cause impaired erections,
but do not generally cause problems with ejaculation once an erection is achieved.

101. Which of the following findings is associated with non-REM (NREM) sleep?
Paradise Institute & Self Study Centre CALL/VIBER: 9818723799 Page 41
PARADISE WEEKLY MODEL TEST – CEE MD/MS BASED POUSH 09

a. Penile tumescence
b. Narcolepsy
c. Dreaming
d. Night terrors

Ans: d
Ref: (Kaplan and Sadock, p 555.)
Night terrors are characterized by a partial awakening accompanied by screaming, thrashing, and autonomic arousal. They
are non-REM sleep events. Increase in blood pressure and heart rate, penile erection, and dreaming are associated with REM
sleep.

Radiology

102. A 2-year-old has an elbow radiograph performed following a fall. Which one of the following epiphyses should
be visible?
a. Capitellum
b. Medial epicondyle
c. None
d. Olecranon

Ans: A
Ref: Grainger & Allison's 5e, p 1613.
▪ The capitellum starts to ossify around 3 months, the radial head and medial epicondyle around 5 years, the trochlea around
8 years and the lateral condyle and the olecranon around 10 years.

103. A 31-year-old woman has a hysterosalpingogram (HSG) as part of a series of investigations for primary
infertility. The HSG shows a single vagina, single cervix but two separate uterine cavities leading to separate
uterine horns. What is the most likely diagnosis?
a. Arcuate uterus
b. Bicornuate uterus
c. Didelphus uterus
d. Septate uterus

Ans: b
Ref: Grainger dr Allison's 5e, p 1203 (Figure 53.2)
▪ A bicornuate uterus is a type of uterine duplication anomaly. It can be classified as a class IV Mullerian duct anomaly.
▪ Overall, congenital uterine anomalies occur in ~1.5% of females (range 0.1-3%). Bicornuate uteri are thought to
represent ~25% (range 10-39%) of Mullerian duct anomalies.
▪ Associations
▪ a longitudinal vaginal septum may be present in ~25% of cases as with other Mullerian duct anomalies, abnormalities
of the renal tract may also be present
▪ Clinical presentation
▪ In most cases, a bicornuate uterus is incidentally discovered when the pelvis is imaged. The most common
symptomatic presentation is with early pregnancy loss and cervical incompetence. Infertility is not usually a problem
with this type of malformation because implantation of the embryo is not impaired.

Paradise Institute & Self Study Centre CALL/VIBER: 9818723799 Page 42


PARADISE WEEKLY MODEL TEST – CEE MD/MS BASED POUSH 09

104. A 70-year-old man with urinary frequency and urge incontinence is referred to the urology outpatient clinic. On
digital rectal examination, his prostate is enlarged and the serum prostate specific antigen (PSA) level is elevated.
What is the most likely finding on IVU?
a. A beaded appearance of the distal urethra
b. A smooth rounded filling defect at the dome of the bladder
c. J-shaped (‘fish-hook’) appearance of the distal ureters
d. Inferior displacement of the ureters by the prostate

Ans: C
Ref Reference: Grainger & Allison's 5e, pp 902-903.
When the prostate enlarges, it pushes up into the base of the urinary bladder. As a result the trigone is elevated which pushes
up the distal ureters (giving a characteristic ‘fish-hook’ appearance).

105. What is not a cause of opacification of a hemithorax?


a. Consolidation
b. Collapse
c. Emphysema
d. Pneumonectomy

Ans : C
Ref: Grainger & Allison’s Radiology, 6th edition, page 190

106. What is not a feature of Kerley A lines?


a. 80 – 100 mm long
b. Angulated occasionally
c. Lie perpendicular to the pleural surface
d. Point medially towards the hilum

Ans: C
Ref: Grainger & Allison’s Radiology, 6th edition, page 388
▪ One of the classical radiographic manifestations of interstitial oedema is thickening of the interlobular septa.
▪ The characteristic Kerley B lines, which represent fluid in the interlobular septa (typically 1–2 mm wide and 30–60
mm long), are only really seen in the subpleural lung, perpendicular to the pleural surface.
▪ By comparison, Kerley A lines are longer (up to 80–100 mm), occasionally angulated and cross the inner two-thirds
of the lung in varying directions but tend to point medially towards the hilum. In left heart failure, septal lines become
visible as they distend with extravascular fluid.

Paradise Institute & Self Study Centre CALL/VIBER: 9818723799 Page 43


PARADISE WEEKLY MODEL TEST – CEE MD/MS BASED POUSH 09

107. What is not a feature of benign breast cysts?


a. Acoustic enhancement
b. Smooth margins
c. Well circumscribed
d. None

Ans: A
Ref: Schwartz’s Surgery, 11th edition, page 571
Second only to mammography in frequency of use for breast imaging, ultrasonography is an important method of resolving
equivocal mammographic findings, defining cystic masses, and demonstrating the echogenic qualities of specific solid
abnormalities.
On ultrasound examination, breast cysts are well circumscribed, with smooth margins and an echo-free center. Benign breast
masses usually show smooth contours, round or oval shapes, weak internal echoes, and well-defined anterior and posterior
margins. Breast cancer characteristically has irregular walls but may have smooth margins with acoustic enhancement

108. What is Thimble bladder associated with?


a. Advanced tuberculous cystitis
b. Early tuberculous cystitis
c. Cystitis cystica
d. Neurogenic bladder

Ans: A
Ref: Craig Hacking, Matt A. Morgan et al.Thimble bladder. Radiopaedia.org
Thimble bladder is a descriptive term for extreme fibrosis and contracture of the bladder walls, resulting in a tiny bladder.
The term is usually used to describe changes from advanced genitourinary tuberculosis.

Dermatology

109. New hair growth is seen in which phase of hair cycle?


a. Anagen
b. Catagen
c. Exogen
d. Telogen

Ans: a
Ref: Ref: Rook’s Dermatology, 9th edition, page 2.10

Paradise Institute & Self Study Centre CALL/VIBER: 9818723799 Page 44


PARADISE WEEKLY MODEL TEST – CEE MD/MS BASED POUSH 09

110. A 48-year-old man presents with recurrent mouth ulcers. Which of the following is not a recognised cause of
mouth ulceration:
a. herpes simplex infection
b. iron deficiency
c. congenital syphilis
d. methotrexate in overdose

Ans: c
Ref: Fitzpatrick’s Dermatology, 9th edition
Causes of mouth ulceration:
• aphthous, idiopathic
• traumatic
• herpes simplex infection
• B12 deficiency, iron deficiency
• coeliac disease, inflammatory bowel disease
• Reiter’s syndrome
• immunodeficiency
• nicorandil, methotrexate, chemotherapy
• oral cancers, metastatic cancer
• syphilitic snail track ulcers
• inflammatory dermatosis e.g. pemphigus.
In this question congenital syphilis is not a cause of mouth ulceration, although secondary syphilis is.

111. What type of human papilloma virus is associated with skin squamous cell carcinoma?
a. 6
b. 11
c. 16
d. 21

Ans: c
Paradise Institute & Self Study Centre CALL/VIBER: 9818723799 Page 45
PARADISE WEEKLY MODEL TEST – CEE MD/MS BASED POUSH 09

Ref: Fitzpatrick’s Dermatology, 9th edition, page 3096

Paradise Institute & Self Study Centre CALL/VIBER: 9818723799 Page 46


PARADISE WEEKLY MODEL TEST – CEE MD/MS BASED POUSH 09

112. What human leukocyte antigen (HLA) is reactive arthritis associated with?
a. B8
b. B27
c. DRw4
d. Dw3

Ans: b
Ref: Ref: Rook’s Dermatology, 9th edition, page 7.7

113. A 23-year-old girl presents with abnormal spoon shaped nails; she has a past history of anaemia, dysmenorrhea
and menorrhagia. You suspect a diagnosis of koilonychia. Which of the following is a recognized cause of
koilonychia:
a. macrocytic anaemia
b. systemic lupus
c. protein losing enteropathy
d. lymphoedema

Ans: B
Ref: Fitzpatrick’s Dermatology, 9th edition, page 2926
• Koilonychia presents with thinned, flat nails with eversion of the lateral nail edges giving a spooned appearance. It is
physiological in children, in adults is commonly due to iron deficiency anaemia.

Paradise Institute & Self Study Centre CALL/VIBER: 9818723799 Page 47


PARADISE WEEKLY MODEL TEST – CEE MD/MS BASED POUSH 09

114. Which cutaneous disease occurs as a result of bacterial toxin of Staphylococcus aureus?
a. Carbuncle
b. Ecthyma
c. Staphylococcal scalded skin syndrome
d. Sycosis

Ans: C
Ref: Rook’s Dermatology, 9th edition, page 26.6

115. Which of the following Fitzpatrick skin type has the greatest tanning ability?
a. Type I
b. Type III
c. Type IV
d. Type VI

Ans: D

Paradise Institute & Self Study Centre CALL/VIBER: 9818723799 Page 48


PARADISE WEEKLY MODEL TEST – CEE MD/MS BASED POUSH 09

Ref: Rook’s Dermatology, 9th edition, page 9.8

Anatomy

116. Goose bump is caused by:


a. Sympathetic Stimulation
b. Parasympathetic Stimulation
c. Somatic Stimulation
d. All of the above

Ans: a
Ref: Moore's Anatomy 7th Edition, Page No: 65

117. Where does lateral corticospinal tract decussate?


a. Dorsal tegmental decussation
b. Ventral tegmental decussation
c. Pyramid
d. Reticular formation

Ans: c
Ref: Gray’s Anatomy for Students, 4th edition, page e48

Paradise Institute & Self Study Centre CALL/VIBER: 9818723799 Page 49


PARADISE WEEKLY MODEL TEST – CEE MD/MS BASED POUSH 09

118. A 31-year-old woman is in her physician’s office for a fitting for an intrauterine contraceptive device. The
physician performs a pelvic examination to ensure that the device is placed in the correct direction. The physical
examination shows that the uterine body is tipped toward the rectum and that the uterine fundus is tipped
anteriorly. Which of the following describes the position of the uterus?
a. Anteverted, anteflexed
b. Anteverted, retroflexed
c. Retroverted, anteflexed
d. Retroverted, retroflexed

Ans: c
Ref: Gray’s Anatomy for Students, 4th edition
“Version” refers to the relation between the uterus and the vagina, whereas “flexion” denotes the relation between the body
and the cervix. Thus, this uterus is retroverted and anteflexed.

119. 39-year-old woman complains of hematuria and significant flank tenderness. She has a history of kidney stones.
A CT scan depicts the abdominal portion of the ureter lying anterior to a muscle. Which of the following is most
likely to be the name of this muscle?
a. Psoas
b. Serratus anterior muscle
c. Obturator muscle
d. External oblique muscle

Ans: a
Ref: Moore KL, Dalley AF. Clinically Oriented Anatomy, 5th ed. Baltimore, MD: Lippincott Williams & Wilkins,
2006:313, 391–4.
The abdominal ureter lies anterior to the psoas muscle.

Paradise Institute & Self Study Centre CALL/VIBER: 9818723799 Page 50


PARADISE WEEKLY MODEL TEST – CEE MD/MS BASED POUSH 09

120. In superior mediastinum, thoracic duct lies on which side of the esophagus?
a. Left
b. Right
c. Anterior
d. Posterior

Ans: a
Ref: Moore's Anatomy 7th Edition, Page No: 166
▪ The esophagus is a fibromuscular tube that extends from the pharynx to the stomach.
▪ The esophagus enters the superior mediastinum between the trachea and vertebral column, where it lies anterior to
the bodies of the T1–T4 vertebrae. The esophagus is usually flattened anteroposteriorly.
▪ Initially, it inclines to the left but is pushed back to the median plane by the arch of the aorta. It is then compressed
anteriorly by the root of the left lung.
▪ In the superior mediastinum, the thoracic duct usually lies on the left side of the esophagus, deep (medial) to the
arch of the aorta.
▪ Inferior to the arch, the esophagus again inclines to the left as it approaches and passes through the esophageal hiatus
in the diaphragm.

121. A 50-year-old man who has diabetes is having difficulty voiding urine. On examination, he has decreased
sensation of the perineal region. Which of the following reflexes is the most likely to be affected?
a. Patellar tendon
b. Achilles tendon
c. Cremaster
d. Anal wink

Ans: d
Ref: Snell RS. Clinical Anatomy by Regions, 8th ed. Baltimore, MD: Lippincott
Williams & Wilkins, 2008:851–77.
The sensory fibers affected are from S2 through S4, which innervate the perineal region and supply the afferent limb of the
anal wink reflex.

122. Cutaneous nerve supply superior to umbilicus is by:


a. T9
b. T10
c. T11
d. T12

Ans: a
Ref: Moore's Anatomy 7th Edition, Page No: 195
Anterior abdominal cutaneous branches of thoraco-abdominal nerves
▪ T7–T9 supply the skin superior to the umbilicus.
▪ T10 supplies the skin around the umbilicus.
▪ T11, plus the cutaneous branches of the subcostal (T12), iliohypogastric, and ilio-inguinal (L1), supply the skin
inferior to the umbilicus.

123. A 15-year-old boy is eating a fish dinner and inadvertently has a bone “caught in his throat.” He complains of
significant pain above the vocal cords. Which of the following nerves is responsible for carrying the sensation for
this pain?
a. Superior laryngeal nerve
b. Recurrent laryngeal nerve

Paradise Institute & Self Study Centre CALL/VIBER: 9818723799 Page 51


PARADISE WEEKLY MODEL TEST – CEE MD/MS BASED POUSH 09

c. Spinal accessory nerve


d. Hypoglossal nerve

Ans: a
Ref: Gray’s Anatomy for Students, 4th edition
The laryngeal mucosa above the vocal cords is innervated by the superior laryngeal nerve, whereas mucosa below the vocal
cords is innervated by the recurrent laryngeal nerve.

124. Postero-inferior surface of scrotum is supplied by:


a. Genital branch of the genitofemoral nerve
b. Anterior scrotal nerves
c. Posterior scrotal nerves
d. Perineal branches of the posterior cutaneous nerve of thigh

Ans: d
Ref: Moore's Anatomy 7th Edition, Page No: 209
The nerves of the scrotum include branches of the lumbar plexus to the anterolateral surface, and branches of the sacral plexus
to the posterior and inferior surfaces:
▪ Genital branch of the genitofemoral nerve (L1, L2): supplying the anterolateral surface.
▪ Anterior scrotal nerves: branches of the ilio-inguinal nerve (L1) supplying the anterior surface.
▪ Posterior scrotal nerves: branches of the perineal branch of the pudendal nerve (S2–S4) supplying the posterior
surface.
▪ Perineal branches of the posterior cutaneous nerve of thigh (S2, S3): supplying the postero-inferior surface.

125. A 47-year-old man complains of right arm weakness and difficulty speaking (expressive aphasia). Which of the
following arteries is most likely affected?
a. Vertebral
b. Posterior cerebral
c. Middle cerebral
d. Anterior cerebral

Ans: c
Ref: Gray’s Anatomy for Students, 4th edition
The middle cerebral artery supplies the temporal and parietal regions that contain Broca area (the speech center).

126. The posterior gastric artery is a branch of which of the following?


a. Hepatic artery
b. Left gastric artery
c. Right gastric artery
d. Splenic artery

Ans: d
Ref: Gray’s Anatomy, 41st edition, page 1117
• A posterior gastric artery supplying the posterior wall of the upper part of the gastric body is commonly present but
there has been a lack of consensus regarding its origin, course and distribution.
• It usually arises from the splenic artery (usually from its mid section), posterior to the body of the stomach, and
ascends behind the peritoneum of the lesser sac towards the fundus to reach the posterior surface of the stomach.

Paradise Institute & Self Study Centre CALL/VIBER: 9818723799 Page 52


PARADISE WEEKLY MODEL TEST – CEE MD/MS BASED POUSH 09

• It may also arise from the left gastric artery or coeliac trunk.

127. A 24-year-old football player receives a blow to the left skull, and the team physician finds weakness of the left
SCM. Which of the following associated findings is most likely to be seen in this patient?
a. Weakness of the masseter muscle
b. Decreased sensation of the ipsilateral face
c. Decreased tearing from the ipsilateral eye
d. Weakness of the trapezius muscle

Ans: d
Ref: BDC Anatomy Volume 2, 6th Edition
The SCM and the trapezius muscle are innervated by the spinal accessory nerve (CN XI), which is at risk in the posterior
triangle of the neck.

Physiology

Paradise Institute & Self Study Centre CALL/VIBER: 9818723799 Page 53


PARADISE WEEKLY MODEL TEST – CEE MD/MS BASED POUSH 09

128. Sulfonylurea treatment in a 53-year-old type 2 diabetic patient causes a fall in the patient’s plasma glucose
concentration to 45 mg/dL. Which of the following is a sign and symptom of hypoglycemia?
a. Bradycardia
b. Dry skin
c. Insomnia
d. Loss of fine motor skills

Ans: d
Ref: Barrett, pp 344-346, 353-354. Kaufman, pp 69-70. Widmaier, p 580.
Hypoglycemia can lead to loss of fine motor skills. Hypoglycemia refers to abnormally low blood glucose levels and is
dangerous because glucose is the primary energy source for brain cells. Dysfunction of the nervous system can lead to
dizziness, headache, mental confusion, convulsion, and loss of consciousness. Increased sympathetic activity can produce
sweating, tachycardia, hunger, and anxiety

129. A 29-year-old woman presents with paroxysmal episodes of headaches, anxiety, and palpitations. The physician
suspects an anxiety disorder, but orders laboratory studies to rule out underlying disease. The laboratory findings
of hypercalcemia and elevated urinary catecholamines suggest the possibility of MEN II. Which of the following is
the hallmark of pheochromocytoma?
a. Dry skin
b. Hypertension
c. Hypoglycemia
d. Lethargy

Ans : b
Ref: (Longo, pp 2269-2273. Barrett, pp 360, 642. Kaufman, pp 94-95.)
The hallmark of pheochromocytoma is either sustained or paroxysmal hypertension. Pheochromocytoma is a rare
catecholamine-secreting tumor of the adrenal chromaffin cells. Patients with the disease often have associated episodes of
sweating, anxiety or nervousness, palpitations, headache, diaphoresis, and hyperglycemia. In adults, approximately 80% of
pheochromocytomas are unilateral and solitary. The 10% rule applies to pheochromocytomas as follows: 10% in adults are
bilateral, 10% are extra-adrenal, 10% are malignant, and 10% are familial, inherited as an autosomal dominant trait either
alone or in combination with MEN II.

130. A 32-year-old alcoholic male with chronic pancreatitis in which more than 90% of pancreatic function is lost.
Which of the following is most likely to be observed?
a. Decreased serum secretin levels
b. Enhanced bile acid micelle formation
c. Increased duodenal pH levels
d. Steatorrhea

Ans: d
Ref: Ref: Guyton & Hall Physiology, 13th edition
This much destruction of the pancreas will lead to a deficiency in the secretion of bicarbonate, digestive enzymes, and islet
cell hormones. Decreased bicarbonate will lead to decreased duodenal pH, which in turn leads to increased serum secretin
levels and to a decreased solu- bility of bile acids in the intestinal lumen. The decreased secretion of lipase along with impaired
micelle formation will lead to fat maldiges- tion and steatorrhea. The loss of adequate insulin secretion will result in elevated
blood glucose levels (diabetes mellitus)..

131. Bainbridge reflex is elucidated by which of the following statements?


a. Fall in atrial pressure decreases the heart rate
b. Fall in ventricular pressure increases the heart rate
c. Rise in atrial pressure increases the heart rate
d. Rise in ventricular pressure decreases the heart rate

Paradise Institute & Self Study Centre CALL/VIBER: 9818723799 Page 54


PARADISE WEEKLY MODEL TEST – CEE MD/MS BASED POUSH 09

Ans: c
Ref: Guyton & Hall physiology, 13th edition, page 223
• An increase in atrial pressure also causes an increase in heart rate, sometimes increasing the heart rate as much as 75
percent. A small part of this increase is caused by a direct effect of the increased atrial volume to stretch the sinus
node; such direct stretch can increase the heart rate as much as 15 percent.
• An additional 40 to 60% increase in rate is caused by a nervous reflex called the Bainbridge reflex. The stretch
receptors of the atria that elicit the Bainbridge reflex transmit their afferent signals through the vagus nerves to the
medulla of the brain. Then efferent signals are transmitted back through vagal and sympathetic nerves to increase
heart rate and strength of contraction.
• Thus, this reflex helps prevent damming of blood in the veins, atria, and pulmonary circulation

132. A 43-year-old woman presents with dysphagia to solids and liquids, bland regurgitation, and diffuse chest pain
of 2 months duration. During this time, she has lost about 8kgs. The patient is scheduled for esophageal imaging
and motility testing. The esophagogram during a barium swallow shows a dilated esophagus with an area of distal
stenosis and esophageal manometry tracings during a wet swallow shows a high lower esophageal sphincter (LES)
opening pressure and uncoordinated peristalsis. These findings are consistent with which of the following
diagnoses?
a. Achalasia
b. Diffuse esophageal spasm
c. Schatzki ring
d. Zenker diverticulum

Ans: a
Ref: Kaufman, pp 110-114. Le, p 324.
• Achalasia is a neurogenic disorder of esophageal motility with the absence of normal peristalsis and impaired
relaxation of the LES in response to deglutition. The barium esophagogram demonstrates a dilated esophagus with a
sharply tapered “birds beak” narrowing of the terminal esophagus. Esophageal manometry shows normal to increased
pressure in the LES with no relaxation in response to swallowing. The etiology is unknown, but achalasia is thought
to arise from scarring (nerve damage) in Auerbach plexus. Treatment is strictly palliative with excellent palliation
available in over 90% of patients. Current pharmacological (nitroglycerin, endoscopic injection of botulinum toxin)
and surgical (balloon dilation, myotomy) therapeutic options are aimed at reducing LES pressure and promoting
esophageal emptying of retained food and liquids.
• Diffuse esophageal spasm is another esophageal motility disorder characterized by frequent nonperistaltic
contractions. The barium esophagogram has a classic “corkscrew” pattern and esophageal manometry shows high
amplitude in the esophagus, sometimes including the proximal esophagus.
• Schatzki ring is a narrow lower esophageal ring-like growth at the gastroesophageal junction associated with
dysphagia to solids only.
• Zenker diverticulum is a pharyngeal or esophageal pouch due to a defect in the muscular wall of the posterior
hypopharynx. The outpouching of the esophagus can be seen on a barium swallow. The clinical signs and symptoms
include sensation of food getting “stuck on the way down,” regurgitation/vomiting of food days after it was eaten,
frequent aspiration, and halitosis.

133. Oral rehydration formulas are noted to facilitate the absorption of sodium chloride and water. This observation
is due in particular because of the inclusion of which of the following?
a. Bile salts
b. Other chloride salts
c. Carbohydrates
d. Proteins

Ans: c
Ref: Guyton & Hall Physiology, 13th edition

Paradise Institute & Self Study Centre CALL/VIBER: 9818723799 Page 55


PARADISE WEEKLY MODEL TEST – CEE MD/MS BASED POUSH 09

Owing to the presence of sodium-coupled glucose transporters in intestinal epithelial cells, sodium and water absorption are
greatly enhanced by the inclusion of carbohydrates. Although amino acid and bile salt absorption also are sodium coupled,
those transporters are not as abundant. Absorption of fatty acids is not sodium coupled.

134. Body temperature is usually lowest at (considering sleep at night and are awake during the day):
a. 6 AM
b. 8 AM
c. 6 PM
d. 10 PM

Ans: a
Ref: Ganong Review of Medical Physiology, Lange; 25th Edition, Page No: 316
• The normal human core temperature undergoes a regular circadian fluctuation of 0.5–0.7°C.
• In individuals who sleep at night and are awake during the day (even when hospitalized at bed rest), it is lowest at
about 6:00 AM and highest in the evenings.
• It is lowest during sleep, is slightly higher in the awake but relaxed state, and rises with activity. In women, an
additional monthly cycle of temperature variation is characterized by a rise in basal temperature at the time of
ovulation.
• Temperature regulation is less precise in young children and they may normally have a temperature that is 0.5°C or
so above the established norm for adults.

135. Smooth muscle can maintain sustained contraction because of which of the following?
a. Continued calcium release from sarcoplasmic reticulum
b. Latch bridge mechanism
c. Myosin ATPase
d. Sustained sympathetic tone

Ans: b
Ref: Ganong’s Review of Physiology, 25th edition, page 116
• As in skeletal muscle and cardiac muscle, calcium plays a prominent role in the initiation of contraction of smooth
muscle. In addition, the lack of troponin in smooth muscle prevents calcium activation via troponin binding. Rather,
myosin in smooth muscle must be phosphorylated for activation of the myosin ATPase.
• In smooth muscle, calcium binds to calmodulin, and the resulting complex activates calmodulin-dependent myosin
light kinase.
• Myosin is dephosphorylated by myosin light chain phosphatase in the cell. However, dephosphorylation of myosin
light chain kinase does not necessarily lead to relaxation of the smooth muscle.
• One mechanism causing this appears to be a latch bridge mechanism by which myosin cross-bridge remains attached
to actin for some time after the cytoplasmic calcium concentration falls. This produces sustained contraction with
little expenditure of energy, which is especially important in vascular smooth muscles.

136. A 45-year-old woman is noted to have fatigue and cold intolerance and is diagnosed with hypothyroidism. Her
physician suspects a pituitary etiology. Which of the following laboratory findings is most consistent with this
condition?
a. Low TSH, low free thyroxine
b. Elevated TSH, low thyroxine
c. Elevated TSH, elevated thyroxine
d. Low TRH

Ans: a
Ref: Ganong Review of Medical Physiology, Lange; 25th Edition

Paradise Institute & Self Study Centre CALL/VIBER: 9818723799 Page 56


PARADISE WEEKLY MODEL TEST – CEE MD/MS BASED POUSH 09

Usually, hypothyroidism is diagnosed by an elevated TSH, because the vast majority of cases of hypothyroidism involve a
primary gland (thyroid) failure. A much less common etiology is a pituitary etiology, which would be reflected by a low TSH.
This would lead to a low free thyroxine but elevated hypothalamic secretion of TRH.

137. Dexamethasone is a synthetic analogue of cortisol. Therapeutically, it can be used to block the effects of
conditions with excessive cortisol secretion. Which of the following is the best description of the mechanism of
action of dexamethasone?
a. Binds to cortisol.
b. Binds to the adrenal gland.
c. Competes for cortisol-binding sites.
d. Inhibits ACTH secretion.

Ans: d
Ref: Guyton & Hall Physiology, 13th edition
Cortisol production and secretion are controlled by ACTH, which is secreted from the anterior lobe of the pituitary gland. A
feedback inhibition loop allows cortisol to inhibit ACTH secretion. The syn- thetic glucocorticoid dexamethasone
effectively and abruptly blocks ACTH secretion, thereby suppressing cortisol secretion.

138. A 17-year-old boy is admitted with a traumatic brain injury, sustained when he fell off his motorcycle. He
develops a fever of 102.2°F (39°C), which is unrelated to an infection or inflammation. The fever is most likely due
to a lesion of which of the following?
a. Anterior hypothalamus
b. Lateral hypothalamus
c. Paraventricular nucleus
d. Posterior nucleus

Ans: a
Ref: Barrett, p 309. Le, p 414.)
• The hypothalamus regulates body temperature. Core body temperature, the temperature of the deep tissues of the
body, is detected by thermoreceptors located within the anterior hypothalamus.
• The anterior hypothalamus also contains neurons responsible for initiating reflexes, such as vasodilation and sweating,
which are designed to reduce body temperature. Heat-producing reflexes, such as shivering, and heat-maintenance
reflexes, such as vasoconstriction, are initiated by neurons located within the posterior hypothalamus.
• The posterior hypothalamus regulates catecholamine secretion. The paraventricular nucleus regulates TSH secretion.
The lateral hypothalamus regulates thirst, and the arcuate nucleus exerts neuroendocrine control of prolactin secretion

139. Which of the following is not reabsorbed at all in the kidneys after being filtered?
a. Bicarbonate
b. Creatinine
c. Glucose
d. Urea

Ans: b
Ref: Guyton & Hall Physiology, 13th edition, page 348

Paradise Institute & Self Study Centre CALL/VIBER: 9818723799 Page 57


PARADISE WEEKLY MODEL TEST – CEE MD/MS BASED POUSH 09

Pathology
140. What produces boot shaped heart?
a. Left atrium hypertrophy
b. Left ventricular hypertrophy
c. Right atrium hypertrophy
d. Right ventricular hypertrophy

Ans: d
Ref: Robbin’s Basic Pathology, 10th edition, page 406
The heart is enlarged and “boot-shaped” as a consequence of right ventricular hypertrophy; the proximal aorta is dilated, while
the pulmonary trunk is hypoplastic.

141. Which of the following conditions is not associated with Helicobacter pylori infection?
a. Peptic ulceration of the duodenum
b. Follicular gastritis
c. Gastric carcinoma
d. Hypertrophic gastropathy

Ans: d
Ref: Robbins and Cotran Pathologic Basis of disease 9/e p768
• Hypertrophic gastropathy refers to hypertrophy of the gastric mucosal folds. There is no association with Helicobacter
infection. The condition actually represents three separate diseases: hyperplasia of the gastric glands secondary to
Zollinger-Ellison syndrome, Menetrier's disease (a disorder of unknown aetiology but possibly due to mucosal over
expression of growth inducing cytokines, frequently asymptomatic), and hypertrophic hypersecretory gastropathy due
to hyperplasia of parietal and chief cells not due to Zollinger-Ellison syndrome, often resulting in peptic ulceration.
• Follicular gastritis refers to the histological pattern often produced by Helicobacter infection, with frequent lymphoid
follicles.

142. Which of the following combinations of signs and symptoms is most consistent with a diagnosis of nephrotic
syndrome?
a. Hematuria, hypertension, and proteinuria
b. Massive proteinuria, edema, and hyperlipidemia
c. Oliguria, hydronephrosis, and abdominal rebound tenderness
d. Painless hematuria, polycythemia, and increased skin pigmentation

Ans: b
Ref: Robbins and Cotran Pathologic Basis of disease 9/e
The two basic glomerular clinical syndromes are the nephrotic syndrome and the nephritic syndrome. The nephrotic syndrome
is characterized by massive proteinuria, which leads to hypoalbuminemia and widespread peripheral edema. Importantly,
these patients also can develop hyperlipidemia (hypercholesterolemia), which is a risk factor for coronary artery disease and

Paradise Institute & Self Study Centre CALL/VIBER: 9818723799 Page 58


PARADISE WEEKLY MODEL TEST – CEE MD/MS BASED POUSH 09

peripheral vascular disease. In contrast, the nephritic syndrome is characterized by hematuria. These patients also can develop
proteinuria, but it is not as massive as the proteinuria seen with the nephrotic syndrome.

143. An infant with ambiguous external genitalia is found to have a XX genotype with external male genitalia and
internal female structures. Physical examination reveals increased blood pressure while laboratory examination
reveals hypernatremia and a hypokalemic alkalosis. Additional laboratory findings include decreased aldosterone,
decreased cortisol, and increased sex steroids. A deficiency of which of the following enzymes is most likely to
produce the clinical findings in this infant?
a. 3-β-dehydrogenase
b. 11-hydroxylase
c. 17-hydroxylase
d. 21-hydroxylase

Ans: b
Ref: Kumar, pp 1152-1154. Rubin, pp 958-961
Ambiguous external genitalia can be seen in some female newborn infants with congenital adrenal hyperplasia (CAH), a
syndrome that results from a defect in the synthesis of cortisol. This leads to excess ACTH secretion by the anterior pituitary
and resultant adrenal hyperplasia. The defect in the
synthesis of cortisol is the result of a deficiency in one of the enzymes in the normal pathway of cortisol synthesis, such as
21-hydroxylase or 11-hydroxylase.
A deficiency of 11-hydroxylase, which is rare, leads to decreased cortisol production and increased ACTH secretion. This in
turn leads to the accumulation of deoxycorticosterone (DOC) and 11-deoxycortisol, both of which are strong
mineralocorticoids. This results in increased sodium retention by the kidneys and hypertension. Patients also lose potassium
and hydrogen ions in the urine, which leads to a hypokalemia metabolic alkalosis. Because of the enzyme block there is
increased formation of 17-hydroxyprogesterone, which is then shunted into the production of testosterone. This may cause
virilism (pseudohermaphroditism) in female infants. That is, XX females with CAH develop ovaries, female ductal structures,
and external male genitalia.

144. Which of the following autoantibodies is most likely to be present in a patient with pernicious anemia?
a. Anticentromere antibodies
b. Antigliadin antibodies
c. Anti-intrinsic factor antibodies
d. Antimitochondrial antibodies

Ans: c
Ref: Robbins Basic Pathology, 10th edition
In contrast to iron deficiency, which causes a microcytic hypochromic anemia, vitamin B12 deficiency produces a macrocytic
anemia. The most common cause of vitamin B12 deficiency in adults is pernicious anemia, which is an autoimmune disease
characterized by the formation of autoantibodies against intrinsic factor and parietal cells of the stomach. A gastric biopsy
from an individual with pernicious anemia will show chronic inflammation with atrophy (chronic atrophic gastritis).

145. Select which of the following bone tumors characteristically arises around the knee joint and gives rise to a soap
bubble appearance on plain film x-ray.
a. Osteoblastoma
b. Chondroblastoma
c. Giant cell tumor
d. Ewing's sarcoma

Ans: c
Ref: Robbins and Cotran Pathologic Basis of disease 9/e p1203
Giant cell tumour is characterized by the presence of multinucleated osteoclast-like giant cells. In most patients, the tumours
are indolent, but they may recur locally in as many as 50% of cases.

Paradise Institute & Self Study Centre CALL/VIBER: 9818723799 Page 59


PARADISE WEEKLY MODEL TEST – CEE MD/MS BASED POUSH 09

146. A 7-year-old boy presents with a rapidly enlarging mass that involves his mandible. A histologic section from
this mass reveals a diffuse proliferation of small noncleaved lymphocytes. In the background are numerous
tingible-body macrophages that have a “starry-sky” appearance when viewed under low magnification. Which of
the following is the most likely diagnosis?
a. Burkitt lymphoma
b. Hodgkin disease
c. Immunoblastic lymphoma
d. Small lymphocytic lymphoma

Ans: a
Ref: Robbin’s Basic Pathology, 10th edition
Burkitt lymphoma is an aggressive, high-grade B-cell lymphoma that is associated with infection by EBV. The
African form of Burkitt lymphoma frequently involves the maxilla or mandible and is more common in children. Burkitt
lymphoma is associated with a characteristic cytogenetic translocation, t(8;14), which involves the c-myc oncogene on
chromosome 18. Histologic sections reveal a diffuse proliferation of malignant small noncleaved lymphocytes within a
background of benign macrophages. These cells impart a “starry sky” appearance to the tissue when viewed under low
magnification.

147. A 60-year-old man has developed truncal obesity, back pain, and skin that bruises easily over the past 5 months.
A chest radiograph shows an ill-defined, 5- cm mass involving the left hilum of the lung. Which of the following
neoplasms is most likely to be present in this patient?
a. Adenocarcinoma
b. Bronchial carcinoid
c. Small-cell carcinoma
d. Squamous cell carcinoma

Ans: c
Ref: Robbins and Cotran Pathologic Basis of disease 9/e p717
• This patient has features of Cushing syndrome, a paraneoplastic syndrome resulting from ectopic corticotropin
production (most often from a pulmonary small-cell carcinoma), which drives the adrenal cortices to produce excess
cortisol.
• Small-cell carcinomas are aggressive tumors that tend to metastasize early. Even when they appear to be small and
localized, they are not or will not remain so. Surgery is not an option for these patients. They are treated as if they
have systemic disease; some chemotherapy protocols afford benefit for 1 year or more, but cure is uncommon.
• Adenocarcinomas and large-cell carcinomas tend to be peripheral neoplasms in the lung, and they are less likely to
produce a paraneoplastic syndrome.
• Bronchial carcinoids tend to be small and are not likely to produce paraneoplastic effects; rarely, they produce
carcinoid syndrome.
• Squamous cell carcinomas can be central and occur in smokers, but they are more likely to produce hypercalcemia.

148. What is responsible for causing the most severe form of aplastic crisis in hereditary spherocytosis?
a. EBV
b. HBV
c. HSV
d. Parvovirus B19

Ans: d
Ref: Robbin’s Basic Pathology, 10th edition, page 445
• The characteristic features are anemia, splenomegaly, and jaundice. The anemia is variable in severity, ranging from
subclinical to profound; most commonly it is moderate in degree. Because of their spherical shape, red cells in
hereditary spherocytosis show increased osmotic fragility when placed in hypotonic salt solutions, a characteristic
that can help establish the diagnosis.

Paradise Institute & Self Study Centre CALL/VIBER: 9818723799 Page 60


PARADISE WEEKLY MODEL TEST – CEE MD/MS BASED POUSH 09

• The course is generally stable but may be punctuated by aplastic crises, the most severe of which are triggered by
parvovirus B19 infection. This virus has a marked tropism for erythroblasts, which undergo apoptosis during viral
replication. Until the immune response controls the infection (usually in 10–14 days), the marrow may be virtually
devoid of red cell progenitors.
• Because of the shortened life span of red cells in hereditary spherocytosis, a lack of red cell production, even for a
few days, results in rapid worsening of the anemia. Blood transfusions may be needed to support patients until the
infection is cleared.

149. What cause of thyrotoxicosis is associated with hyperthyroidism?


a. de Quervain thyroiditis
b. Factitious thyrotoxicosis
c. Iodine-induced hyperthyroidism
d. Subacute lymphocytic thyroiditis

Ans: c
Ref: Robbins Basic Pathology, 10th edition, page 756

Pharmacology
150. A woman deemed at high risk of postmenopausal osteoporosis is started on alendronate. Which of the following
is this representative bisphosphonate’s main mechanism of action?
a. Activates vitamin D
b. Directly forms hydroxyapatite crystals in the bone
c. Provide supplemental phosphate, which indirectly elevates serum Ca2+
d. Reduces the number and activity of osteoclasts in bone

Ans : d
Ref : Brunton, pp 1666–1668; Craig, pp 758–760; Katzung, pp 721, 727–730.
Whether used for osteoporosis (prevention or management, men or women, idiopathic or drug-induced) or Paget’s disease of
the bone, bisphosphonates exert their effects on osteoclasts and osteoblasts. The drug is incorporated into bone. When drug-
containing bone is resorbed by the osteoclasts, osteoclast function (and, so, subsequent
bone resorption) is inhibited. The bisphosphonates also recruit osteoblasts, which then produce a substance that further inhibits
osteoclast activity

Paradise Institute & Self Study Centre CALL/VIBER: 9818723799 Page 61


PARADISE WEEKLY MODEL TEST – CEE MD/MS BASED POUSH 09

151. A patient with an endocrine disorder develops lactic acidosis, and nearly dies, as a result of an uncommon but
serious adverse response to therapy with an “endocrine” drug. Which of the following was the most likely cause of
this severe problem?
a. Insulin glargine, prescribed for Type 1 diabetes mellitus
b. Levothyroxine, prescribed to maintain euthyroid status following thyroidectomy
c. Metformin, prescribed for Type 2 diabetes mellitus
d. Propylthiouracil, prescribed for hyperthyroidism

Ans: c
Ref : Brunton, pp 1638–1639; Craig, pp 773–774; Katzung, p 708.)
Metformin, a common drug for managing many patients with Type 2 diabetes mellitus, is a very well-tolerated and effective
drug. This biguanide not only lowers circulating glucose levels, but suppresses appetite, thereby lowering dietary caloric
intake. One of the main problems associated with metformin is the development of lactic acidosis—admittedly rare but
potentially fatal adverse response to this drug and associated with none of the other drugs listed as answer choices.

152. An 8-year-old child is diagnosed with type I diabetes. Insulin deficiency results in which of the following?
a. Decreased glycogenolysis
b. Decreased lipolysis
c. Increased protein synthesis
d. Ketogenesis

Ans: d
Ref: Katzung Pharmacology, 14th edition, page 209
Insulin deficiency promotes a series of reactions in the body that are like adaptation to food deprivation or starvation. The
initial response is to maintain the plasma glucose concentration through increased glycogenolysis and hepatic
gluconeogenesis. Without the insulin-dependent control, there will be a decrease in the rate of glu- cose utilization because
of decreased permeability in insulin- dependent tissues such as muscle and adipose tissue. Limiting glucose permeability to
adipose tissue will inhibit triglyceride formation, resulting in increased release of FFA into the circulation. Hepatic oxi- dation
of FFA will produce ketone bodies that will enter the circulation, providing alternative energy sources for tissues, and reduce
glucose uti- lization further. Insulin promotes muscle protein synthesis, and in its absence protein catabolism will prevail,
with a release of glucogenic precursors into the circulation for hepatic gluconeogenesis.

153. Which drug has little or no role in diastolic heart failure?


a. Angiotensin receptor blocker
b. Beta blocker
c. Calcium channel blockers
d. Digoxin

Ans: d
Ref: Katzung Pharmacology, 14th edition, page 222

Paradise Institute & Self Study Centre CALL/VIBER: 9818723799 Page 62


PARADISE WEEKLY MODEL TEST – CEE MD/MS BASED POUSH 09

154. A 75-year-old man has a fever of 104°F. He develops a cough that produces blood-tinged sputum with gram-
positive cocci in clusters. A chest x-ray shows increased density in the right upper lobe. Which of the following
penicillins is likely to fail to treat this infection adequately?
a. Cloxacillin
b. Dicloxacillin
c. Oxacillin
d. Ticarcillin

Ans: d
Ref: (Brunton, pp 1131t, 1133; Craig, pp 529–530.)
Ticarcillin, which is quite similar to carbenicillin in many clinically relevant ways, has a high degree of potency against
Pseudomonas and Proteus organisms, but is inactivated by penicillinase produced by various bacteria, including most
staphylococci. Oxacillin, cloxacillin, nafcillin, and dicloxacillin are all
penicillinase-resistant and are effective against staphylococci.

155. Predominantly NA reuptake inhibitor is:


a. Amoxapine
b. Trazodone
c. Moclobemide
d. Paroxetine

Ans: a
Ref: KD Tripathi Essentials of Medical Pharmacology; 7th Edition, Page No: 454
• Reversible inhibitors of MAO-A (RIMAs): Moclobemide, Clorgyline

Paradise Institute & Self Study Centre CALL/VIBER: 9818723799 Page 63


PARADISE WEEKLY MODEL TEST – CEE MD/MS BASED POUSH 09

• Tricyclic antidepressants (TCAs)


o NA + 5-HT reuptake inhibitors: Imipramine, Amitriptyline, Trimipramine, Doxepin, Dothiepin,
Clomipramine
o Predominantly NA reuptake inhibitors: Desipramine, Nortriptyline, Amoxapine, Reboxetine
• Selective serotonin reuptake inhibitors (SSRIs): Fluoxetine, Fluvoxamine, Paroxetine, Sertraline, Citalopram,
Escitalopram, Dapoxetine
• Serotonin and noradrenaline reuptake inhibitors (SNRIs): Venlafaxine, Duloxetine
• Atypical antidepressants: Trazodone, Mianserin, Mirtazapine, Bupropion, Tianeptine, Amineptine, Atomoxetine

156. Glycoprotein IIb/IIIa receptor antagonist is:


a. Clopidogrel
b. Abciximab
c. Tranexamic acid
d. Ticlopidine

Ans: b
Ref: KD Tripathi Essentials of Medical Pharmacology; 7th Edition, Page No: 629
Antiplatelet Drugs – Aspirin, Dipyridamole
P2Y12 Receptor Blockers - Ticlopidine, Clopidogrel, Prasugrel
GPIIb/IIIa Antagonists – Abciximab, Eptifibatide,Tirofiban

157. Where does acetazolamide act at?


a. Proximal convoluted tubule
b. Distal convoluted tubule
c. Thin descending limb
d. Thick ascending limb

Ans: a
Ref: Katzung Pharmacology, 14th edition, page 255

Paradise Institute & Self Study Centre CALL/VIBER: 9818723799 Page 64


PARADISE WEEKLY MODEL TEST – CEE MD/MS BASED POUSH 09

158. A patient consumes an excessive dose of theophylline and develops toxicity in response to the drug. Which of the
following is the most likely consequence of this?
a. Drowsiness progressing to sleep and then coma
b. Hepatotoxicity
c. Paradoxical bronchospasm
d. Seizures

Ans: d
Ref: Brunton, pp 728–730; Craig, p 463; Katzung, pp 325–327
• Theophylline, a methylxanthine, is a caffeine–like drug that is becoming outmoded as therapy for asthma in most
adolescents and adults. A low margin of safety, extreme dependence on adequate liver function (for metabolism),
susceptibility to numerous clinically significant drug interactions, and a lack of airway anti-inflammatory activity, are
among the reasons why without proper dosage adjustments and monitoring it is all too easy, and common, for blood
levels to fall into subtherapeutic ranges or, as we see here, into toxic ranges.
• The earliest signs and symptoms of excess involve CNS stimulation (jitteriness, tremors, difficulty sleeping, anxiety).
As blood levels rise the CNS is increasingly stimulated. Seizures may occur, and when they do the inability to breathe
during the seizures is the main cause of death.
• Theophylline tends to cause tachycardia, increases of cardiac contractility and, potentially, tachyarrhythmias.
Theophylline is not hepatotoxic; it does not cause paradoxical bronchospasm, even when serum levels are very high
or truly toxic.

159. A 35-year-old woman undergoes a thyroidectomy for papillary serous thyroid cancer. The surgeon suspects that
the parathyroid glands have been removed. Which of the following findings is most likely to be seen in the patient
1 week postoperatively?
a. Coma
b. Constipation
c. Esophagitis

Paradise Institute & Self Study Centre CALL/VIBER: 9818723799 Page 65


PARADISE WEEKLY MODEL TEST – CEE MD/MS BASED POUSH 09

d. Muscle spasms and tetany

Ans: d
Ref: KD Tripathi Essentials of Medical Pharmacology; 7th Edition
Removal of the parathyroid glands may lead to hypocalcemia. Symptoms include nerve paresthesias, muscle spasms, and
tetany. A physical sign is the Trousseau sign, the development of carpal spasm when the blood pressure cuff is inflated for
about 2 to 3 minutes. The Chvostek sign is twitching of facial muscle when the facial nerve is percussed lightly anterior to
the ear. Severe hypocalcemia can lead to seizures, laryngospasm, and lethargy. The other answers refer to symptoms or signs
of hypercalcemia.

Biochemistry

160. A noncompetitive inhibitor of an enzyme does which of the following?


a. Decreases Vmax
b. Increases Vmax
c. Decreases Km and decreases Vmax
d. Increases Km with no or little change in Vmax

Ans: a
Ref: (Murray, pp 70-83. Scriver, pp 4571-4636.)
• In contrast to competitive inhibitors, noncompetitive inhibitors are not structural analogs of the substrate.
Consequently, noncompetitive inhibitors bind to enzymes in locations remote from the active site. For this reason,
the degree of inhibition is based solely upon the concentration of inhibitor, and increasing the substrate concentrations
does not compete with or change the inhibition.
• Therefore, unlike the increase in Km seen with competitive inhibition, in noncompetitive inhibition, Vmax decreases
while Km usually remains the same. While competitive inhibitors can be overcome at a sufficiently high concentration
of substrate, noncompetitive inhibition is irreversible.

161. Which of the following is an energy-requiring step of glycolysis?


a. Glucokinase
b. Phosphoglycerate kinase
c. Pyruvate kinase
d. Phosphohexose isomerase

Ans: a
Ref: (Murray, pp 187-190. Scriver, pp 1433-1436.)
• Glucokinase catalyzes the conversion of glucose to glucose 6-phosphate in the energy, requiring first step of
glycolysis. ATP is also required in the conversion of fructose 6-phosphate to fructose 1,6-bisphosphate by
phosphofructokinase. ATP is generated in the conversion of 1,3-bisphosphoglycerate to 3-phosphoglycerate by
phosphoglycerate kinase and in the conversion of phosphoenolpyruvate to pyruvate by pyruvate kinase
• No energy is required when lactate dehydrogenase converts pyruvate to lactate (a molecule of NADH is converted to
NAD+ or when phosphohexose isomerase converts glucose 6-phosphate to fructose 6-phosphate.

162. Which of the following would be least likely to stimulate vasopressin release from the posterior pituitary?
a. Dehydration
b. Angiotensin II
c. Atrial stretch receptors
d. Aldosterone

Ans: d
Ref: Harper’s Biochemistry, 30th edition
Aldosterone acts independent of vasopressin to increase water resorption by the kidney, by stimulating the insertion of ion
transporters into the membrane of distal colon and kidney distal tubule.
Paradise Institute & Self Study Centre CALL/VIBER: 9818723799 Page 66
PARADISE WEEKLY MODEL TEST – CEE MD/MS BASED POUSH 09

163. The anticodon region is an important part of the structure of:


a. r-RNA
b. m-RNA
c. hn-RNA
d. t-RNA

Ans: d
Ref: Chattarjea Textbook of Biochemistry 8th Edition, Page No: 248

164. What is measured by Kjeldahl’s method?


a. Carbon
b. Hydrogen
c. Nitrogen
d. Oxygen

Ans : c
Ref: Satyanarayana Biochemistry, 4th edition, page 44
• The content of nitrogen, an essential component of proteins, on an average is 16%.
• Estimation of nitrogen in the laboratory (mostly by Kjeldahl’s method) is also used to find out the amount of protein
in biological fluids and foods.

165. In a normal premenopausal woman, which one of the following is stimulated by progesterone?
a. Release of gonadotropin-releasing hormone by the pituitary
b. Ovulation
c. Development of the endometrium in preparation for possible pregnancy
d. Uterine contraction

Ans: c
Ref: Satyanarayana Biochemistry, 4th edition
Progesterone is secreted by the corpus luteum under the influence of LH. Together with estrogen, it promotes the thickening
and main- tenance of the endometrium. Progesterone inhibits GnRH release, uterine contraction, and follicle development.

166. The rate limiting step in cholesterol biosynthesis is:


a. Squalene synthetase
b. Mevalonate kinase
c. HMG-CoA synthetase
d. HMG-CoA reductase

Ans: d
Ref: Chattarjea Textbook of Biochemistry 8th Edition, Page No: 438
• 7-α-hydroxylase activity is also enhanced by cholesterol of endogenous and dietary origin and regulated by insulins,
glucagon, glucocorticoids and thyroid hormones.
• The controlling enzyme for cholesterol biosynthesis is HMG-CoA reductase.
Paradise Institute & Self Study Centre CALL/VIBER: 9818723799 Page 67
PARADISE WEEKLY MODEL TEST – CEE MD/MS BASED POUSH 09

• Activities of both these probably change in parallel and undergo similar ‘diurnal’ variation. Both the enzymes may
exist in ‘active’ and ‘inactive’ form which may be regulated by phosphorylation/dephosphorylation mechanisms.

167. The most common cause of hypercortisolism is which of the following?


a. Adrenal tumor that secretes excess cortisol and mineralocorticoid hormones.
b. Lung tumor that secretes excess ACTH that leads to excess cortisol in the blood.
c. Administration of synthetic cortisol by physician
d. Pituitary adenoma that secretes excess ACTH, leading to excess cortisol in the blood

Ans: d
Ref: Satyanarayana Biochemistry, 4th edition, page 48 – 49
While all of the choices can lead to hypercortisolism, about two- thirds of all cases are due to a pituitary adenoma that secretes
excess ACTH, leading to excess cortisol in the blood.

Microbiology
168. Comma shaped Gram-negative organism is:
a. V cholerae
b. E coli
c. Shigella
d. Salmonella

Ans: a
Ref: Katzung’s Basic & Clinical Pharmacology; Lange; 14th Edition, Page No: 253
Upon first isolation, V cholerae is a comma-shaped, curved rod 2–4 μm long (Figure 17-1). It is actively motile by means of
a polar flagellum. On prolonged cultivation, vibrios may become straight rods that resemble the Gram-negative enteric
bacteria.

169. Which one of the following hepatitis virion is the largest?


a. Hepatitis A
b. Hepatitis B
c. Hepatitis C
d. Hepatitis D

Ans: c
Ref: Jawetz microbiology, 27th edition, page 496

Paradise Institute & Self Study Centre CALL/VIBER: 9818723799 Page 68


PARADISE WEEKLY MODEL TEST – CEE MD/MS BASED POUSH 09

170. Human B19 virus targets:


a. Erythroid progenitor cells
b. Platelets
c. WBC
d. Lymph

Ans: a
Ref: Jawetz’s Medical Microbiology; Lange; 27th Edition, Page No: 445
▪ Parvoviruses are small viruses with single-stranded DNA genomes.
▪ Human B19 virus targets erythroid progenitor cells.
▪ B19 is associated with erythema infectiosum (fifth disease), transient aplastic crisis, pure red cell aplasia, and hydrops
fetalis (most commonly in early pregnancy).
▪ Human bocaviruses have been associated with acute respiratory disease and gastroenteritis in children, but causation
has not been proven.
▪ Human B19 and bocaviruses are difficult to grow; laboratory diagnosis depends on serology and molecular assays.

171. Vector for Trypanosoma cruzi is:


a. Tsetse fly
b. Kissing bug
c. Sandfly
d. Mosquito

Ans: b
Ref: Jawetz’s Medical Microbiology; Lange; 27th Edition, Page No: 714

172. Similar to rotavirus, which of the following viral agents is also a nonenveloped RNA virus known to cause
gastroenteritis diarrhea in young children?
a. Calicivirus
b. Paramyxovirus
c. Parainfluenza virus
d. None of the above

Ans: a
Ref: Ananthanarayan Microbiology, 10th edition
• Like rotaviruses, caliciviruses are nonenveloped RNA viruses that cause watery diarrhea, especially in children
• Paramyxoviruses are enveloped RNA viruses that cause childhood respiratory and exanthemous infections;
• Parainfluenza viruses are enveloped RNA viruses which cause respiratory infections such as croup, bronchiolitis,
and pneumonia in children;
• Coxsackie viruses are nonenveloped RNA viruses that cause nonspecific respiratory tract infections, febrile rashes,
and meningitis.

173. Which of the following is normal flora of skin:

Paradise Institute & Self Study Centre CALL/VIBER: 9818723799 Page 69


PARADISE WEEKLY MODEL TEST – CEE MD/MS BASED POUSH 09

a. Staphylococcus epidermidis
b. Propionibacterium species
c. Acinetobacter species
d. All of the above

Ans: d
Ref: Jawetz’s Medical Microbiology; Lange; 27th Edition, Page No: 170
Normal flora of Skin:
▪ Staphylococcus epidermidis
▪ Staphylococcus aureus (in small numbers)
▪ Micrococcus species
▪ α-Hemolytic and nonhemolytic streptococci (eg, Streptococcus mitis)
▪ Corynebacterium species
▪ Propionibacterium species
▪ Peptostreptococcus species
▪ Acinetobacter species
▪ Small numbers of other organisms (Candida species, Pseudomonas aeruginosa, etc)

174. A 56-year-old woman presents to an ER The woman is homeless and extremely dirty. She is also very
dehydrated and the duty nurse notices lice in her hair and clothing. The patient reported high fever, severe
headaches, and muscle pain. A petechial rash was observed all over her body. After 72 hours in the hospital, the
woman died. A Gram-stain of the organism showed a weakly staining Gram-negative bacterium that could not be
grown in vitro. What organism was the most likely cause of her infection?
a. Chlamydia trachomatis
b. Rickettsia prowazekii
c. Rickettsia rickettsii
d. Rickettsia typhi

Ans: b
Ref: (Murray [2009], pp 427-433. Murray [2013], Ch 41; Ryan, Ch 40.)
• The source of the woman’s infection is Rickettsia prowazekii. Her disease is louse-borne typhus. Louse-borne typhus
is a disease with a mortality of approximately 30%.
• Chlamydia trachomatis is obviously incorrect because the woman does not have trachoma, which is primarily a
disease of the eye or genital tract.
• Rickettsia rickettsii causes Rocky Mountain spotted fever, which is the most common Rickettsia disease in the United
States, but is spread by ticks. Rickettsia typhi causes endemic typhus or
• murine typhus and is spread by fleas. Therefore, it is important to know the arthropod vectors that spread Rickettsia
diseases to be able to tell them apart.

175. An examination of sputum for a suspected case of fungal infection may reveal hyphae in which of the following?
a. Aspergillosis
b. Cryptococcosis
c. Histoplasmosis
d. Sporotrichosis

Ans: a
Ref: Ananthanarayan Microbiology, 10th edition
Cryptococcosis, histoplasmosis, paracoccidioidomycosis and sporotrichosis are all caused by dimorphic fungi. At 37oC
(98oF), the yeast form predominates. Aspergillosis, on the other hand, is caused by an organism that produces only hyphae
(no-yeast component).

Community Medicine
Paradise Institute & Self Study Centre CALL/VIBER: 9818723799 Page 70
PARADISE WEEKLY MODEL TEST – CEE MD/MS BASED POUSH 09

176. Chernobyl nuclear explosion accident occurred on 26th April 1986. It resulted in emission of:
a. Methyl isocyanate (MIC)
b. Union carbide
c. Ur235, Po210
d. I131, Cs134, Cs137, Sr90

Park 20/e p706, Park 21/e p742, Park 22/e p746]


• Chernobyl nuclear explosion accident occurred on 26th April, 1986 in Russia (now Ukraine)
– It resulted in emission of I131, Cs134, Cs137, Sr90
• Chernobyl nuclear explosion accident is the ‘largest accidental release of radioactive material in the history of nuclear power’
• It is the only instance so far of level 7 on the International Nuclear Event Scale for nuclear accidents
• World’s worst man-made disaster is Bhopal gas Tragedy, 3rd December 1984:
– Methylisocyanate (MIC) gas leaked from Union Carbide pesticide plant in Bhopal, India
– It resulted in resulting in the death of about 3,000 people according to the Indian Supreme Court
– Fukushima Daichii Tragedy, 11 March 2011: I131, Cs134, Cs137.

177. Dengue shock syndrome is characterized by the following except:


a. Hepatomegaly
b. Pleural effusion
c. Thrombocytopenia
d. Decreased hemoglobin.

Ans. d
Ref. Park 21/e p226-27, Park 22/e p227-28-29
• Classical dengue fever (DF):
– Also known as ‘breakbone fever’
– Clinical features: High grade fever (biphasic curve) with chills, intense headache, muscle and joint pains, retro-orbital
pain, photophobia, colicky pain, abdominal tenderness, skin rash
• Dengue hemorrhagic fever (DHF): Severe form of DF, caused by infection with more than one dengue virus type
– Incubation period: 4 – 6 days
– Clinical features: Features of DF plus
o Rash less common
o Rising hematocrit value (> 20% of baseline)
o Moderate-to-marked thrombocytopenia (< 1 lac/ mm3)
o Hepatomegaly
o Positive tourniquet test: > 20 petechiae per sq. inch
– Diagnosis of DHF: Fever + hemorrhagic manifestations + thrombocytopenia + hemoconcentration or rising
hematocrit
• Dengue shock syndrome (DSS):
– Diagnosis of DSS: DHF + shock [rapid and weak pulse, narrow pulse pressure (< 20 mm Hg)/ hypotension, cold
clammy skin, restlessness

178. All of the following are advantages of case control studies except
a. Relative risk can be calculated
b. Odds ratio can be calculated
c. Useful in rare diseases
d. Quick results are obtained

Ans: a
Ref: Park's Textbook of Preventive and Social Medicine; 24th Edition; Page No: 41

Paradise Institute & Self Study Centre CALL/VIBER: 9818723799 Page 71


PARADISE WEEKLY MODEL TEST – CEE MD/MS BASED POUSH 09

Since incidence rates are essential for calculation of relative risk, which are not provided by case control studies as
appropriate denominator, or population at risk is not available, relative risk cannot be calculated with the help of case control
studies. It can only be determined from a cohort study.

179. In a case control study of smoking and lung cancer, which of the following can be a possible conclusion?
a. Smoking is a cause of lung cancer
b. Lung cancer is commoner in smokers than non- smokers
c. If smoking is stopped, the number of cases of lung cancer will decrease
d. Smoking is associated with lung cancer

Ans: d
Ref: Park's Textbook of Preventive and Social Medicine; 24th Edition;
Since case control studies proceed backward from effect to cause. they mainly indicate statistical
association between exposure and occurrence of disease. Statistical association does not imply causation, however, strength
of this association can be measured by odds ratio

180. Diabetes mellitus is best diagnosed by:


a. Fasting blood sugar (FBS) > 100 mg/dl and post-prandial blood sugar (PPBS) >140
b. FBS >125 mg/dl and PPBS >199 mg/dl
c. HbA1c = 5.5%
d. FBS > 70 md/dl

Ans: b
WHO GUIDELINES FOR DIAGNOSIS OF DIABETES MELLITUS
• Fasting plasma glucose level: > 126 mg/dL (> 7 mmol/L)
• 2-hour venous plasma glucose in Glucose tolerance test: > 200 mg/dL (> 11.1 mmol/L)
• Casual plasma glucose: > 200 mg/dL (> 11.1 mmol/L)
• Glycated haemoglobin: > 6.5%

181. WHO STEPS is used for:


a. Communicable diseases
b. Non- communicable diseases
c. Immuno- deficient diseases
d. Auto-immune diseases

Ans. b
Ref. World Health Organisation
• STEP wise approach to surveillance (STEPS): Is a simple, standardized method by WHO for surveillance
• Is of two types
• STEP wise approach to chronic disease risk factor surveillance
• STEP wise approach to Stroke surveillance
• Comprises of 3 steps:

Paradise Institute & Self Study Centre CALL/VIBER: 9818723799 Page 72


PARADISE WEEKLY MODEL TEST – CEE MD/MS BASED POUSH 09

Research methodology
182. If professional judgment concerning a primary interest tends to be unduly influenced by a secondary interest,
this is known as:
a. Vested interest
b. Unlawful interest
c. Conflict of interest
d. Financial interest

Ans: c
"Conflict of interest" in the field of medical research has been defined as "a set of conditions in which professional judgment
concerning a primary interest (such as a person's welfare or the validity of research) tends to be unduly influenced by a
secondary interest (such as financial gain)."

183. A researcher draws unbiased sample of 100 adults and finds that their mean weight is 72 kg with a standard
deviation of 1.5. 95% CI for of weight of adults shall be:
a. 66 and 78 kg
b. 69 and 75 kg
c. 70.5 and 73.5 kg
d. None of the above

Ans. b
Ref. Simple Biostatistics by Indrayan & Indrayan, 1/e p141 and Methods in Biostatistics by Mahajan, 7/e p92; Park
21/e p788, Park 22/e p792]
• Confidence interval (CI):
– Is the interval within which a parameter value is expected to lie with certain confidence levels, as could be revealed
by repeated samples
– Is the ‘range that is likely to contain the population mean when so obtained for repeated samples’
– A narrow CI is always preferable: as it tells more precisely what the population might be mean BUT also it will have
higher chances of not containing the population mean
1. Larger the sample size, narrower is CI
2. Smaller the SD (s), narrower is CI
95% CI for population mean = Mean  2SD ( + 2σ)
In the given question, n = 100 adult, Mean weight () = 72 kg, Standard deviation (σ) = 1.5
95% CI for of weight
= Mean  2SD (+ 2σ)
= 72  2 (1.5)
= 72 – 3, 72 + 3 = 69, 75 kg.

Paradise Institute & Self Study Centre CALL/VIBER: 9818723799 Page 73


PARADISE WEEKLY MODEL TEST – CEE MD/MS BASED POUSH 09

184. Histogram is used as method of group presentation for:


a. Qualitative data
b. Quantitative continuous data
c. Quantitative data- discrete type
d. Norminal data

Simple Biostatistics by Indrayan & Indrayan, 1/e p104 and Methods in Biostatistics by Mahajan, 6/e p20, 7/e p18; Park
21/e p783-85, Park 22/e p787-89]
Histogram:
– Is graphical presentation for ‘continuous quantitative data’
– Continuous groups are marked on x-axis (abscissa) while frequencies are marked on y-axis (ordinate).
Quantitative data Qualitative data
Histogram Bar diagram
Frequency polygon Pie/Sector diagram
Frequency curve Pictogram/Picture diagram
Line chart/graph Map diagram/Spot map
Cumulative frequency diagram (Ogive)
Scatter/Dot diagram

185. During an investigation on the pandemic, researchers found that the virus can divide every 20 minutes.
Beginning with a single individual, how many viruses will be there in the population if there is exponential growth
for 3 hours?
a. 18
b. 440
c. 512
d. 1024

Ans: c
P = C (1 + r)n
where, P = Final value ; C = Initial value; r = fraction increase; n = no. of times increase
In the given question, A bacterium can divide every 20 minutes and exponential growth is for 3 hours,
Thus, C = 1, r = 1 (it doubles every time); n = 3 hours/20 minutes = 9 times
Thus P = C (1 + r)n = 1 (1 + 1)9 = 29 = 512
……..
One Other Simple Way of Doing It-
• Given information: A virus can divide every 20 minutes (3/hour)
• Thus in 3 hours, it will total divide 3/hour × 3 hours = 9 times
• Thus 9 times multiplication will successively yield 2, 4, 8, 16, 32, 64, 128, 256, 512 virus.
• So at end of 9 hours, there will be 512 virus.

Forensic Medicine

Paradise Institute & Self Study Centre CALL/VIBER: 9818723799 Page 74


PARADISE WEEKLY MODEL TEST – CEE MD/MS BASED POUSH 09

186. In terminal illness, care to provide relief of physical pain, discomfort or emotional and psychological suffering is
termed as:
a. Palliative care
b. Passive euthanasia
c. Physician-assisted suicide
d. Active euthanasia

Ans: a
Ref: Gautam Biswas Forensic Medicine, 3rdEdition, Page No: 26
Palliative care: The provision of reasonable medical and nursing procedures for the relief of physical pain, discomfort or
emotional and psychological suffering as well as providing food and water in terminally ill patients.

187. Which of the following is not consistent with airborne transmission over droplet transmission:
a. It refers to the presence of microbes within droplet nuclei
b. Can remain in the air for long periods of time
c. Can be transmitted to others over distances greater than 1 m
d. Particles > 15μm in diameter are the source

Ans: d
Ref: Official WHO website {Accessed on April 8, 2020] Modes of transmission of the COVID-19 virus
• Respiratory infections can be transmitted through droplets of different sizes: when the droplet particles are >5-10 μm
in diameter they are referred to as respiratory droplets, and when then are <5μm in diameter, they are referred to as
droplet nuclei. According to current evidence, COVID-19 virus is primarily transmitted between people through
respiratory droplets and contact routes. In an analysis of 75,465 COVID-19 cases in China, airborne transmission was
not reported.
• Droplet transmission occurs when a person is in in close contact (within 1 m) with someone who has respiratory
symptoms (e.g., coughing or sneezing) and is therefore at risk of having his/her mucosae (mouth and nose) or
conjunctiva (eyes) exposed to potentially infective respiratory droplets. Transmission may also occur through fomites
in the immediate environment around the infected person.8 Therefore, transmission of the COVID-19 virus can occur
by direct contact with infected people and indirect contact with surfaces in the immediate environment or with objects
used on the infected person (e.g., stethoscope or thermometer).
• Airborne transmission is different from droplet transmission as it refers to the presence of microbes within droplet
nuclei, which are generally considered to be particles <5μm in diameter, can remain in the air for long periods of time
and be transmitted to others over distances greater than 1 m.
• In the context of COVID-19, airborne transmission may be possible in specific circumstances and settings in which
procedures or support treatments that generate aerosols are performed; i.e., endotracheal intubation, bronchoscopy,
open suctioning, administration of nebulized treatment, manual ventilation before intubation, turning the patient to
the prone position, disconnecting the patient from the ventilator, non-invasive positive- pressure ventilation,
tracheostomy, and cardiopulmonary resuscitation.
• There is some evidence that COVID-19 infection may lead to intestinal infection and be present in faeces. However,
to date only one study has cultured the COVID-19 virus from a single stool specimen. There have been no reports of
faecal−oral transmission of the COVID-19 virus to date.

188. Krönlein shot is an injury caused to:


a. Skull
b. Chest
c. Abdomen
d. Lower limb

Ans: a
Ref: Gautam Biswas Forensic Medicine, 3rdEdition, Page No: 224

Paradise Institute & Self Study Centre CALL/VIBER: 9818723799 Page 75


PARADISE WEEKLY MODEL TEST – CEE MD/MS BASED POUSH 09

Krönlein shot is a very rare injury of the skull caused by a high-velocity bullet. In this close-range shot, there is bursting of
the skull and laceration of the dura mater with complete evisceration of the brain. The autopsy could not determine with
certainty the appearance of the primary gunshot wound, and the entrance and exit wounds.

189. A person was found dead with bluish green frothy discharge at the angle of mouth and nostrils. Probable cause
can be:
a. Arsenic poisoning
b. Copper poisoning
c. Mercury poisoning
d. Lead poisoning

: Gautam Biswas Forensic Medicine 3rd Edition, Page No: 506, 507
Chronic Copper Poisoning
• Cause- Chronic copper toxicity may occur from eating acidic foods cooked in uncoated copper cookware, or from
exposure to excess copper in drinking water or food contaminated with verdigris, or other environmental sources.
• It may also occur among workers using copper and its salts due to inhalation of copper dust or fumes— welders may
develop metal fume fever.
Signs and Symptoms
• Green or purple line on the gums, a constant metallic taste, nausea, dyspepsia, vomiting and diarrhea with colicky
pain.
• Giddiness and headache.
• Laryngitis and bronchitis.
• Renal damage.
• General signs of progressive emaciation, viz. anemia, malaise and debility.
• Peripheral neuritis with wrist drop or foot drop and atrophy of muscles.
• Copper dust may cause inflammation of the conjunctiva and ulceration of the cornea.
• Skin becomes jaundiced. Urine and perspiration become green.
• Bronzed diabetes may be present

190. Vicarious responsibility pertains to:


a. Patient’s, contribution towards negligence
b. Hospitals contribution towards patient’s damage
c. Responsibility for actions of a colleague
d. Responsibility of senior for actions of junior

Ans: d
Ref: Gautam Biswas Forensic Medicine, 3rdEdition, Page No: 21
Vicarious Liability/Respondent Superior
An employer is responsible not only for his own negligent act, but also for the negligent act of his employees by the principle
of ‘respondeat superior’ (Latin, ‘let the master answer’), if three conditions are satisfied:
• There must be an employer-employee relationship
• The employee’s conduct must occur within the scope of his employment
• Incident must occur while on the job. It also called the ‘Master-Servant Rule’.
In medical practice, usually, the principal doctor becomes responsible for any negligence of his assistants (both medical and
para-medical). Both may be sued by the patient, even though the principal has no part in the negligent act

Mandatory CPD Topics


191. Total budget allocated for ministry of Health and population in Nepal in FY 2023-24 is:
a. NPR 103.09 billion
b. NPR 98.08 billion
c. NPR 122.02 billion
d. NPR 83.99 billion

Paradise Institute & Self Study Centre CALL/VIBER: 9818723799 Page 76


PARADISE WEEKLY MODEL TEST – CEE MD/MS BASED POUSH 09

Ans: d
Ref: www.mof.gov.np
Budget for health in FY 2023-24:
Health budget: 83.99 billion to the Ministry of Health and Population

192. Which of the following strategies has been shown to improve recovery from the post-cardiac arrest syndrome?
a. Early echocardiography
b. Cooling of core temperature to 28°C to 30°C
c. Maintaining 02 saturation of 100%
d. Cooling of core temperature of 32°C to 34°C

Ans: d
Ref: Bernard S. Hypothermia after cardiac arrest: expanding the therapeutic scope. Crit Care Med. 2009;37 (7
suppl):S227-S233..
Therapeutic hypothermia to core temperatures of32°C to 42°C for 24 to 48 hours has been shown to improve
neurological outcomes in patient following V-fib arrests. Cooling of patients to 28°C to 30°C is associated with increased risk
of arrhythmia without additional improvement in neurological outcomes. Maintaining PA02 of 100% could result in hyperoxia
and has been shown to increase mortality. Even though maintaining euvolemia improves post-cardiac arrest patient outcomes,
the use of Swan-Ganz catheter goal-directed therapy has not been proven to have survival benefits in these patients.

193. Which of the following is important in patient–physician communication?


a. Documentation based communication
b. Enforced decision making
c. Sharing of personal information
d. Patient-centered interviewing

Ans: d
Physicians’ ability to effectively and compassionately communicate information is key to a successful patient–physician
relationship. The current health care environment demands increasing clinical productivity and affords less time with each
patient, which can impede effective patient–physician communication.
The use of patient-centered interviewing, caring communication skills, and shared decision making improves patient–
physician communication.
Involving advanced practice nurses or physician assistants may improve the patient’s experience and understanding of her
visit. Electronic communication with established patients also can enhance the patient experience in select situations.

194. A 33-year-old man was admitted to the ICU after having been found comatose in his home with a suicide note
and an empty bottle of aspirin (30 count). His salicylate level returns at 111 mg/dL and his serum pH is 7.01. What
is the best treatment plan for this patient?
a. Octreotide can be considered if the patient does not respond to IV dextrose administration.
b. Sodium bicarbonate infusion should be initiated with a goal serum pH 7.45 to 7.55.
c. Hemodialysis should be initiated to enhance elimination and correct the acidosis.
d. N-acetylcysteine treatment should be started, and serial liver function should be monitored during the treatment.

Ans: c
Ref: Olson K. Poisoning and Drug Overdose. 5th ed. San Francisco, CA: McGraw Hill; 2004.
Hemodialysis is indicated in salicylate toxicity with a serum level above 100 mg/dL with profound acidosis. Sodium
bicarbonate is the mainstay of treatment with salicylate toxicity; however, the goal of treatment is to alkalinize the urine to
enhance salicylate elimination. Therefore, the goal of sodium bicarbonate therapy is to maintain a urinary pH of 7.5 to 8.0.
Octreotide and N-acetylcysteine are not indicated in salicylate toxicity.

195. A 55-year-old man involved in RTA is unresponsive and is intubated immediately the scene. On arrival in the
ER, he responds to painful stimulation. His systolic BP is 60 mm Hg, his HR is 140 bpm, his neck veins are

Paradise Institute & Self Study Centre CALL/VIBER: 9818723799 Page 77


PARADISE WEEKLY MODEL TEST – CEE MD/MS BASED POUSH 09

distended, and his breath sounds are absent on the left side. Immediate management should involve which of the
following?
a. Insertion of a central venous line on the right side
b. Insertion of an 18-gauge needle in the left second intercostal space
c. Pericardiocentesis
d. Peritoneal lavage

Ans: b
The patient has tension pneumothorax, as evidenced by distended neck veins and absent breath sounds. Increased intrathoracic
pressure interferes with venous return to the heart, resulting in shock.
Immediate management should be insertion of a large-bore needle in the left second intercostal space, followed by insertion
of a chest tube. In a trauma patient, venous access should be achieved by inserting two large-bore (16-gauge) angiocatheters
in the cubital veins. Insertion of a central venous line on the right side should not be done, because it carries the risk of
producing pneumothorax in the opposite side.

196. Doctor is ethically and legally bound to maintain confidentiality between physician and patient except:
a. Child abuse
b. Gunshot or stabbing injuries
c. Patient is harm to others or self
d. All of the above

Ans: d
Confidentiality between physician and patient is generally absolute
Exceptions
• Suspicion of child/elder abuse
• Gunshot or stabbing injuries must be reported to the police
• Communicable disease must be reported
• Patient is a harm to others or self: Tarasoff decision
• No alternative means exists to warn others
• Patient waves right to privacy e.g., for insurance purposes

197. Components of rational use of medicines as per WHO definition does not include?
a. Appropriate to clinical needs
b. Doses that meet their own individual requirements
c. Lowest cost
d. Minimal side effects

Ans: d
As per the WHO (1985), the definition of rational use of medicines – “Patients receive medications appropriate to their clinical
needs, in doses that meet their own individual requirements, for an adequate period of time, and at the lowest cost to them and
their community.”

198. 1 adolescent male was struck by a vehicle and lying on the roadside, paramedics during the basic life support
techniques, pulse is checked at:
a. Brachial
b. Radial
c. Carotid
d. Umbilical

Ans: c
Summary of recommended basic life support techniques:

Paradise Institute & Self Study Centre CALL/VIBER: 9818723799 Page 78


PARADISE WEEKLY MODEL TEST – CEE MD/MS BASED POUSH 09

199. Competence of a patient is:


a. Medical decision
b. Legal decision
c. Local guardian decision
d. CDO decision

Ans: b
Capacity is a medical term
Competence is a legal term
Competent patients have the right to refuse medical information and medical treatment(s)

200. Heinrich’s safety pyramid is:


a. Unsafe acts or near misses lead to minor injuries and, over time, to a major injury
b. Patient safety is more prioritized than health care worker safety
c. Health care worker safety is more prioritized than patient safety
d. Auditing of patient risk factors

Ans: a
Heinrich’s safety pyramid
• Developed in 1931, Heinrich’s safety pyramid theorised that unsafe acts or near misses lead to minor injuries and,
over time, to a major injury. The accident pyramid proposes that for every 300 near misses there are 29 minor
injuries and one major injury.
• Risk assessment, which is a step-in risk management that calculates the value of risk related to a situation or hazard,
has shown us that what prevents patients from being hurt is not only by reducing the number of mistakes but rather
by increasing the number of defences set up against the consequences of mistakes.
• The key message is that near misses provide the best data about the reliability of safety systems. It is, therefore,
most important to report near misses as well as adverse events to ensure that defences against adverse events are
built and sustained.

NEXT MD/MS BASED MODEL EXAM


❖ COMING SATURDAY (ONLINE & OFFLINE)
❖ COMING TUESDAY (ONLINE ONLY)

BEST WISHES

Paradise Institute & Self Study Centre CALL/VIBER: 9818723799 Page 79


PARADISE WEEKLY MODEL TEST – CEE MD/MS BASED POUSH 09

Paradise Institute & Self Study Centre CALL/VIBER: 9818723799 Page 80

You might also like